Sie sind auf Seite 1von 150

1.

The school districts plan to build a new, expanded school building in Townsville is ridiculous.
While there is enough money in the budget to pay for a larger building, there is none left over to
pay the salaries of additional teachers, so the additional new classrooms would sit empty and
unused.

Assumption Hitting CR.


The conclusion drawn to the left is most called into question by which of the following?
A. A larger school building will allow for expanded extracurricular facilities.
B. The new proposed school building would have space for nineteen more classrooms than does
the current building.
C. Paying for the larger school building would force the school district to cut back its bus
services.
D. The condition of a school building correlates with many other factors, including low dropout
rates and high standardized test scores.
E. In the current school building, there is not space for all the classes, and many classes are held
in portable trailers on the school grounds
Second Attempt: Time: 1:53; OA: IMO: E

OA : E

2. Ever since Mayor May Knotts publicity campaign for New York Citys new express subway
service began last year, car traffic in the citys crowded uptown decreased 10%. In that same
year, there was an equivalent increase in the number of people riding subways uptown. Clearly,
the mayors publicity campaign persuaded many people to ride subways rather than drive their
cars.
Prediction base answer Weaken CR
Which of the following, if true, casts the most serious doubt on the passages conclusion?
A. New car prices have risen about 12% over the last year.
B. The mayor commutes uptown to City Hall daily on the subway.
C. Over the last year, pothole repairs have closed many lanes on roads leading uptown, which
were previously used by commuters.
D. The total number of subway trains that come uptown in the mornings has stayed exactly the
same as it was one year ago.
E. Surveys reveal that city residents who had been riding available subways for years were no
more satisfied with subway service than they were a year ago, before the publicity campaign
began.
Second Attempt: Time: 1:53; OA: IMO: C; Take: different reason for change of ppls mind to
ride subways than drive
OA C

3. Use of traditional pesticides on edible crops such as spinach, carrots, and grapes
entails a risk that the consumer will not properly wash such foods and will become
sick as a result of pesticide contamination. But not using the pesticides means that
there is a chance that such crops will be consumed by pests, and will never make it
out of the fields. Therefore, continuing to use traditional pesticides is the only way
to ensure that people are able to obtain healthy fruits and vegetables
Only one way type Weaken CR

The argument to the left is most called into question by which of the following, if true?
A. There are effective non-traditional pesticides that are far less harmful to humans than those
traditionally used.
B
Not eating fruits and vegetables at all is more dangerous to one's health than eating those
contaminated with trace amounts of pesticide.
C
Traditional pesticides dangers can be minimized with careful handling.
D
People usually rinse fruits and vegetables before eating them.
E
Buying organically grown fruits and vegetables is prohibitively expensive for many families.
My Take: A. This is the credited response. If non-traditional pesticides can be used and are less
harmful to humans, then they are a better alternative than either of the choices presented by the
author.
B. This answer choice strengthens the argument, by supporting the authors point that pesticide is
the better of the two bad options.
C. This answer strengthens the argument, by minimizing the dangers of traditional pesticides.
D. This answer strengthens the argument, by showing that people can and do minimize their
risks.
E. This answer choice eliminates a possible alternative for most families, but does not weaken
the argument.
OA : A

4. Abandoned nests of a distinctive design and containing feathers from the nowextinct Adolfo bird were found in Missibama, and ornithologists estimated that those
nests were about 75 years old. Based on this, the commonly held belief was that the
Adolfo bird first migrated to Missibama at around that time. However, more
recently, the same kind of unique nests were found, and were estimated to be over
100 years old. Therefore, the Adolfo bird must have come to Missibama at least 100
years ago

Prediction base answer Weaken CR


The conclusion to the left is most called into question by which of the following, if true?
A. The Adolfo bird was hunted into extinction for because its subtle flavor made it a delicacy in
some parts of the world.
B. Missibamas many forests make it an excellent location for birdwatching.
C. The Adolfo bird appears to have come north to Missibama from Belize.
D. Adolfo birds are believed to have been incapable of building their own nests, and instead
killed birds of other species and adopted their nests.
E. Adolfo birds were well suited for the humid, hot climate of Missibama.
Second attempt: Time: 1:29; OA: IMO: D
OA : D

5. Advertisement: For bad breath, five out of six dentists treat their patients with
Eliminex. So for the most effective bad breath treatment, choose Eliminex

Alternative explanation
Which of the following, if true, most weakens the advertisement's argument?
A. Some bad breath treatments will also help whiten teeth.
B. Dentists often choose products based on cost and ease of use.
C. Eliminex is available over the counter.
D. Some dentists will not prescribe Eliminex to patients who may be irritated by its key
ingredient.
E. Studies have shown that Eliminex is a much more effective treatment for bad breath than
many comparable products.

6. In the barren African desert, water is available at an oasis, but each person or
pack animals daily water consumption is strictly rationed: travelers may take only

enough drinking water for themselves and their animals for seven days. But trees
grow along the oasis rim in a narrow circle and also consume its water. Clearly,
therefore, if the travelers cut down those trees, they will have more water for
drinking

Prediction base answer Weaken CR


Which of the following, if true, most seriously weakens the argument?
A. Trees around the oasiss rim provide shade, and even protect the oasis from extreme loss of
water through evaporation from the sun and wind.
B. Frequent travelers who use the oasis will demand a larger drinking water supply before
investing in the cost of cutting the trees around the oasis.
C. The species of trees on the rim of the oasis have evolved to grow only in soil where the roots
are always wet.
D. Even if the trees were cut down, the circles of land around the oasis, on which they grow, is
too rocky to have any use.
E. The rationing and designated use of drinking water is designed to let travelers and pack
animals take enough water to get to the next oasis, without depleting it for other travelers.
Take away: Choice A is the correct answer. If trees protect the oasis from evaporation, removing
the trees may not result in more water.
Choice B is not the correct answer. This statement is irrelevant. The argument is about the
effectiveness of the plan in increasing the water supply, not the cost.
Choice C is not the correct answer. This statement is irrelevant. The argument proposes cutting
all species.
Choice D is not the correct answer. This statement is irrelevant. The argument is about
chopping trees to get more water, not about using the land on the rims.
Choice E is not the correct answer. This statement is irrelevant. The justification for rationing
does not affect the argument that chopping trees will produce more water.

7. Farmers of spinach are reeling from crop contamination, which caused a recent
disease outbreak. In response to consumer protests, spinach farmers have claimed
that all leafy vegetable crops have had similar contamination problems

Switch From part (Spinach) to Whole (All leafy)


Which of the following, if true, would most undermine the farmers' claim?
A. Sales of spinach following the disease outbreaks have plummeted, causing economic setbacks
among farmers of the crop.
B. Even washing contaminated produce would not make it suitable for consumption.
C. Escarole and red cabbage are other kinds of leafy vegetables, but their consumers have had no
outbreaks of disease that would follow from contamination.
D. Similar spinach contamination has been discovered in South America, a situation in which the
crop's irrigation water was polluted by animal refuse
E. Legislation has been proposed for greater sanitary regulation of produce farmers.
OA : C

8. Some environmentalists question the prudence of exploiting features of


the environment, arguing that there are no economic benefits to be gained
from forests, mountains, or wetlands that no longer exist. Many
environmentalists claim that because nature has intrinsic value it would be

wrong to destroy such features of the environment, even if the economic


costs of doing so were outweighed by the economic costs of not doing so.
Which one of the following can be logically inferred from the passage?
(A) It is economically imprudent to exploit features of the environment.
(B) Some environmentalists appeal to a noneconomic justification in
questioning the defensibility of exploiting features of the environment.
(C) Most environmentalists appeal to economic reasons in questioning the
defensibility of exploiting features of the environment.
(D) Many environmentalists provide only a noneconomic justification in
questioning the defensibility of exploiting features of the environment.
(E) Even if there is no economic reason for protecting the environment, there
is a sound noneconomic justification for doing so.
Answer: B

CR type : Inference
Type : Modifier based CR such as some many..
Concentrate on modifier.
My Explanation : Some env. Question the pru. Of explo. Features of env. = no eno. Benefit
that not exist.
Many env. : Nat. has intrinsic value so its wrong to destroy them.

A) It is economically imprudent to exploit features of the environment.


Nowhere it is said so, the argument has claims.. no conclusion..

(B) Some environmentalists appeal to a noneconomic justification in


questioning the defensibility of exploiting features of the environment.
Yup correct
(C) Most environmentalists appeal to economic reasons in questioning the
defensibility of exploiting features of the environment.
Most env. Doesnt give eco. Reason ..wrong

(D) Many environmentalists provide only a noneconomic justification in


questioning the defensibility of exploiting features of the environment.
This choice is wrong because of the "only".
The passage claims that certain environmentalists have advanced a noneconomic justification, but never says that this is the ONLY justification
advanced by those environmentalists.
In fact, note that, REGARDLESS OF THE QUESTION, this CANNOT be the
correct answer IF (b) is also an available option. (reason: if (d) is true, then
(b) MUST also be true, since it's a weaker claim about the same thing.
therefore, if (d) is true, then (b) is a fortiori also true. since you can't have
two correct answers, (d) can't possibly be correct.

(E) Even if there is no economic reason for protecting the environment, there
is a sound noneconomic justification for doing so.
Out of scope

9. Economist: In the interaction between producers and consumers, the only


obligation that all parties have to act in the best interests of their own side.
And distribution of information about product defects is in the best interests

of the consumer, So consumers are always obligated to report product


defects they discover, while producers are never obligated to reveal them.
Which one of the following is an assumption required by the economist's
argument?
A. It is never in the best interests of producers for a producer to reveal a
product defect.
B. No one expects producers to act in a manner counter to their own best
interests
C. Any product defect is likely to be discovered by consumer
D. A product defect is more likely to be discovered by a consumer than by a
producer
E. The best interests of consumers never coincide with the best interests of
producers
Answer: A
Explanation:
Argument: In the interaction between producers and consumers, the only
obligation that all parties have to act in the best interests of their own side.
Conclusion: So consumers are always obligated to report product defects
they discover, while producers are never obligated to reveal them.
Assumption: It is never in the best interests of producers for a producer to
reveal a product defect

10. The workers at Bell Manufacturing will shortly go on strike unless the
management increases their wages. As Bells president is well aware,
however, in order to increase the workers wages, Bell would have to sell off
some of its subsidiaries. So, some of Bells subsidiaries will be sold.
The conclusion above is properly drawn if which one of the following is
assumed?
(A) Bell Manufacturing will begin to suffer increased losses.
(B) Bells management will refuse to increase its workers wages.
(C) The workers at Bell Manufacturing will not be going on strike.
(D) Bells president has the authority to offer the workers their desired wage
increase.
(E) Bells workers will not accept a package of improved benefits in place of
their desired wage increase

OA : C
The workers will go on strike if their wages are not raised; their wages can
only be raised by selling subsidiaries, right? Only one of two things can
happen here:
1) Subsidiaries ARE SOLD so wages can be raised, and therefore the workers
DO NOT strike.
2) Subsidiaries ARE NOT SOLD, the wages are not raised, and therefore the
workers DO go on strike.
The conclusion in the passage, though, says that subsidiaries will be sold
(choice 2 in my list) -- Bell Manufacturing does not want its workers to go on
strike, and that is the assumption built into the conclusion.

11. Advertisement:Anyone who exercises knows from firsthand experience


that exercise leads to better performance of such physical organs as the
heart and the lungs, as well as to improvement in muscle tone. And since
your brain is a physical organ, your actions can improve its performance, too.
Act now. Subscribe to Stimulus: read the magazine that exercises your brain.
The advertisement employs which one of the following argumentative
strategies?
(A) It cites experimental evidence that subscribing to the product being
advertised has desirable consequences.
(B) It ridicules people who do not subscribe to Stimulus by suggesting that
they do not believe that exercise will improve brain capacity.
(C) It explains the process by which the product being advertised brings
about the result claimed for its use.
(D) It supports its recommendation by a careful analysis of the concept of
exercise.
(E) It implies that brains and muscle are similar in one respect because they
are similar in another respect.
OA : E
We don't actually learn HOW this product is meant to exercise our brain. So, there is
no explanation of "process". On this basis, we can eliminate C.
Choice E correctly describes the method of argument. Brains and muscles are
similar in one respect--they are both "physical organs". Thus, the ad concludes they
must be similar in another respect: our actions can improve their performance.

12. Under the influence of todays computer-oriented culture, publishing for


children has taken on a flashy new look that emphasizes illustrations and
graphic design; the resulting lack of substance leads to books that are shortlived items covering mainly trendy subjects. The changes also include more
humorous content, simplification of difficult material, and a narrower focus
on specific topics.
Which one of the following is most strongly supported by the information
above?
A. The inclusion of humorous material and a narrower focus detract from the
substance of a childrens book.
B. The substance of a childrens book is important to its longevity.
C. Children of the computer generation cannot concentrate on long,
unbroken sections of prose.
D. Children judge books primarily on the basis of graphic design.
E. The lack of substance of a childrens book is unlikely to be important to its
popularity.

OA : B

Type CR : Inference
Just concentrate on Results : lack of sub -> short live d items.

13. Last year, AutoSafe sold a large number of new automobile insurance
policies and luxury cars were stolen at a particularly high rate, resulting in a
record number of theft-related insurance claims submitted to AutoSafe. This
year, Autosafe will no longer sell insurance policies to owners of luxury cars.
Therefore, unless Autosafe sells a record number of policies to owners of
non-luxury cars, fewer theft-related claims will be submitted this year.
Which of the following is an assumption on which the argument relies?
(A) Each year, luxury cars are stolen at a higher rate than are non-luxury
cars.
(B) Customers who purchased policies last year are no more likely to have
their cars stolen this year than they were last year.
(C) This year, the theft rate of non-luxury cars will not be substantially higher
than the theft rate of non-luxury cars last year.
(D) This year, the number of stolen cars recovered by AutoSafe policyholders
will be greater than the number of stolen cars recovered by policyholders
last year.
(E) The number of automobile thefts that affected AutoSafe policyholders last
year was not nearly as high as the number of thefts that affected nonpolicyholders.
Cr Type: Assumption
Style: Scope change from luxury to non-luxury.
Assumption: 1. No. of non-luxury cars thefts not reduced.
2. Rate on non-luxury cars theft no increased sufficiently.
3. No. of Luxury cars owner are as active as that of No. of nonluxury cars owner
i.e. no change in claims.
OA : C -- Match to point 2

14. To prevent its tenants from relocating their offices, a building


management company has hired a customer service employee who monitors
tenant satisfaction. When the new employee determines that a tenant may
be considering relocation, the management company offers the tenant
benefits, such as reduced rent, to induce the tenant to stay. Since hiring the
new employee, the management company has cut in half the number of its
tenants who have relocated. The new employee, therefore, is increasing the
management companys profits by reducing tenant turnover.
Which of the following is an assumption on which the argument depends?
(A) The presence of the new employee increases the satisfaction of all
tenants, not just those who may be considering relocation.
(B) The costs of the new employee and the benefits offered to induce tenants
to stay do not outweigh the financial benefit of reducing tenant turnover.
(C) The new employee was hired on the recommendation of a consultant who
has consistently increased the profits of other building management
companies.
(D) The new employee was hired before the summer months, when most
companies choose to relocate their offices.
(E) The cost of refurbishing an empty office before seeking a new tenant
outweighs the cost of providing benefits that induce a current tenant to stay.
CR: Assumption type
Style: Addition data.
My Assumptions: 1. New employee knows about tenant satisfaction.
2. Half the no. of the tenants provide high amount.
3. ---Cost of new employee and the cost of satisfying
tenant < fin. Benefit of reducing tenant turnover.
OA : B

15. When a laptop computer needs maintenance, it is often shipped to a


central location for service, then shipped back when the service is complete.
In order to reduce the risk of additional problems caused by jostling in
transit, MP Tech has hired part-time technicians to provide maintenance
service on MP Tech laptops in cities across the country. Providing local
technicians costs MP Tech approximately the same amount as paying for
shipping as well as employing a large centralized staff.
Which of the following, if true, is further evidence that using local technicians
will help MP Tech achieve its goal?
(A) Moving maintenance services away from a centralized location makes it
less economical to provide phone support.
(B) Customers who take their laptop computers to local technicians pack
their computers extremely carefully to avoid causing further problems in
transit.
(C) MP Tech laptop computers are unique, so their technicians require special
training.
(D) Purchasers of MP Tech laptop computers tend to use their machines more
than average, and they require more frequent service than purchasers of
other computer brands.
(E) Many of the local technicians MP Tech has hired have experience working
as technicians at MP Techs centralized location
CR Type: Strengthen Argument.
Style: Conclusion.
My take: To reduce the risk of probs. during transit, MP tech plan to hire part
time tech. in city.
So check whether some other thing is causing the other problems during
transit.
OA : B

16. Although the pesticides used in nearby fields are the primary cause of
pollution in the Ellenville River, researchers believe that increased
automobile traffic in the immediate vicinity is a contributing cause, since
nearly all water samples taken from the river show signs of contamination
from vehicular emissions. Such emissions are not typically considered a
major source of water pollution, but researchers hypothesize that vehicular
emissions are the source of more pollution in the Ellenville River than are
pesticides.
Which of the following, if true, most strongly supports the researchers
hypothesis?
(A) Research has shown that there is more pollution from vehicular emissions
in the Ellenville River than in any other river within a 500 miles radius.
(B) The water filtration system that produces drinking water for Ellenville is
specifically designed to protect against the pesticides used in nearby fields.
(C) Water samples from the Ellenville River taken fifty years ago show
evidence of pollution from vehicular emissions.
(D) In laboratories without the most sophisticated diagnostic tools, it is
difficult to differentiate between types of water pollutants.
(E) The diseases caused by the different forms of water pollution cause
different health problems, and the illnesses caused by vehicular emissions
are much more prevalent
in Ellenville than are those caused by pesticides
CR Type: Strengthen argument.
Style: Comparison.

My take: Conclusion: emission from car is not a major src. Of pollution ,but it
is src of more pollution that are pesticides.
Take: Try to find an answer which talks about the comparison
So find answer which helps to find the above reason
A. Talks about river being more polluted not effecting conclusion.
B. Talks about the remedy taken for pesticides pollution. wrong out of
scope.
C. It doesnt matter how long is this happening and just talks about
pollution from vehicle
D. Neither strengthen nor weaken.
E. OA answer

17. Carnigan International Airport was once the busiest airport in the region,
but two major airlines have relocated to nearby airports, reducing the
number of flights in and out of Carnigan by more than half. The gates at
Carnigan were built more than thirty years ago and cannot accommodate the
largest modern aircraft. In an effort to bring in more business, Carnigan
officials plan to build dozens of gates to accommodate modern aircraft and
offer reduced-rate leases to airlines willing to make long-term commitments.
Which of the following, if true, most threatens the plans likelihood of
success?
(A) Most of the airlines operating out of nearby airports have long-term
leases on gates at the other airports.
(B) The existing gates at Carnigan rent at rates that are, on average, much
lower than the proposed rates for the new gates to be built there.
(C) Because of highway congestion and changing development patterns that
make Carnigan inconvenient to access, airline customers prefer to depart
from other airports.
(D) Of the 18 airlines that serve Carnigan or nearby airports, none serve
more than one airport in the area.
(E) If the proposed addition is completed, Carnigan will be the largest airport,
as measured both by gates and square footage, in the region.

http://www.beatthegmat.com/doubt-in-cr-t67694.html#304929

18. Situation: Geologic evidence strongly suggests that at some point in the
next decade, the La Maria volcano will erupt. News of the potential eruption
is likely to cause tourists to stay away from destinations in La Maria County.
Goal: The La Maria Chamber of Commerce wishes to maintain the present
high rate of tourism in La Maria County.
Proposal for consideration: Develop an advertising campaign that
emphasizes the safety of La Maria destinations in the case of a volcano
eruption.
In light of the situation, which of the following, if true, most strongly argues
that adopting the proposal would be an ineffective way of achieving the
goal?
(A) Tourists who visit La Maria County rate safety as one of their top priorities
in choosing a vacation destination.
(B) All over the world, there are active volcanoes near areas whose
economies depend on tourism.
(C) Very few visitors to La Maria County participate in volcano-related
activities such as camping and hiking.
(D) Most potential vacationers to La Maria County are not aware that the
volcano is active, and would discover that it is through advertisements that
emphasize the areas safety.

(E) Compared to tourist destinations that are not near active volcanoes, La
Maria County is an inexpensive place for a family to vacation.
CR Type: Strengthen
OA: IMO: D
Strongly argues makes this sound like a strengthen question, but the
word ineective tells you were looking to weaken the proposal. Since the
passage is broken up like this, its worth trying to paraphrase the
argument. Basically: The volcano may erupt, which might make tourists
skittish, but we want to keep tourists coming, so we should advertise that the
area is still safe.
Choice (A) strengthens the argument by providing a solid reason for the
advertisements. (B) is irrelevant, as there is no mention on whether tourists
visit those areas when volcanoes are likely to erupt, and what prompts them
to do so. (C) is also irrelevant, as we dont know whether the safety issues
extend to tourists who are not on the mountain itself.
Choice (D) explains why advertising may not be a good idea. If they arent
aware of the safety issues, they wont stay away; if the advertisements make
them aware of the safety issues, they may still come, but they certainly
wouldnt be more likely to do so. (E) is irrelevant, as the argument has
nothing to do with cost. Choice (D) is correct

19. At present Satellex Radio provides only music stations. However, many
Satellex subscribers listen to talk radio stations as well, and they would
prefer that Satellex offered talk radio as well. Moreover, subscribers to radio
services that provide talk radio usually subscribe for longer periods of time
than those who subscribe to music-only services. Therefore, if Satellex added
talk radio stations, its profits would increase.
The argument is vulnerable to criticism on the grounds that it gives reason to
believe that it is likely that
(A) Satellexs large subscriber base would appeal to talk radio hosts, making
it easy for Satellex to hire quality talent for its talk radio stations.
(B) Most talk radio hosts broadcast on advertiser-supported radio stations
that listeners can access without paying a subscription fee.
(C) Satellexs costs would rise by adding several talk-radio stations, while
many of the listeners who want talk radio are already Satellex subscribers.
(D) Talk radio listeners generally spend more time on a single radio station
than do listeners to music radio.

(E) If Satellex added talk radio stations, many of Satellexs music radio hosts
would switch to talk-radio stations, making the music stations less appealing.
OA: C
CR Type: Weaken
Style: Hitting Evidence
Tutor: This problem plays on the three part formula profits = revenuesexpenses.
When first reading the argument, I found that the premises are that there is
a demand for radio talk shows, which I presumed meant that Satellex can
increase its revenues. The conclusion (profits will increase) immediately hit
me as too far fetched, as the argument ignores the third part of the formula the expenses. In order to reach the conclusion that the move will be
profitable, the argument must assume that the costs involved are smaller
than the added revenues. Many GMAT questions play on this, and it's
important to recognize this dynamic - the "leap" form revenues to profits,
which ignores expenses.
Since the question stem basically asks which of the following would weaken
the argument, I was looking for something like C - something that points out
the costs ignored by the argument. C did take me slightly by surprise by
indicating that even the revenues side may not increase, since a large part of
the target audience is already a subscriber, but that actually makes C an
even stronger weakening of the argument - if Satellex increases costs, but
does not generate new revenues, then the whole move is even less likely to
be profitable. I believe C is the answer
My Take: More sub. Will subscribe for longer time to radio-services that
provide talk radio than those who subscribe to music only radio Increase
in profits
: If company if not going to increase the cost of existing subscriber and as
there are many subscriber. The company will be in loss.
: The quality of talk radio is worse than that of other talk radio. Thus few
subscribers enroll in this plan.
: Adding Talk radio stations add lot to a cost of the Radio, which is more than
the cost obtain from long term committed subscriber.
POE:
A. Though quality is at hand an issue, it never say that quality may affect
the profit. O.F.S
B. What other do is not important though there is possibility that people
switch to Free subscription fee bcoz quality can be the issue with adv.supported radio stations.
C. Correct.

D. Spending more time than required is not important or leads to more


profit.
E. This would strengthen the argument.

20. At a start-up company, hiring is often done by the companys founder.


Since founders are often experts in fields far removed from the expertise
they are looking for in employees, hiring decisions in early-stage start-ups
can result in poor matches between the company and the employee. Enough
poor matches can make it nearly impossible for such a company to succeed
in the marketplace.
Which of the following, if true, points to the most serious flaw of a start-up
founders plan to outsource hiring to a firm that specializes in matching
employees and early-stage start-up companies?
(A) It is not well understood why founders often make poor choices when
selecting new employees for their companies.

(B) If a founder is successful in hiring an employee that is a good match for


his company, he or she will not necessarily be successful the next time a new
employee must be hired.
(C) While founders tend to make decisions quickly; firms that handle hiring
decisions for start-up companies interview more candidates and take longer
to select an employee.
(D) Firms that handle hiring decisions for start-ups rarely use interviewers
who have the expertise that a start-up company is looking for in a given
employee.
(E) The average starting salary paid to employees selected by outside firms
is considerably higher than the average starting salary paid to employees
hired directly by a
start-ups founder.
OA: D
CR Type: Weakening
Style: Hitting the assumption.
My Take: Conclusion: a start-up founders plan to outsource hiring to a firm
that specializes in matching employees and early-stage start-up companies.
Assumption: That firm will select a proper match for the company.
POE
A. It talks about founder not about the firm O.F.S
B. Same as A.
C. Interviewing more candidates and taking longer time are not the issue
at hand. -> O.F.S
D. Correct. (weaken the assumption)
E. Salary is not important.

21. In Manila, Phillipines, one of the largest shopping malls in the world, the
recently-opened SM Mall of Asia, is threatening the viability of other supermalls. In Manila, but its appearance has
had a positive effect on many small businesses in the area. The SM Mall o f
Asia has few of the same attractions as other Manila super-malls, but it has
become of the largest tourist destinations in the Phillipines.

Which of the following contributes most to an explanation of the difference


between the SM Mall of Asias effect on other super-malls and small
businesses?
(A) Tourists often visit the Phillipines because of the wide variety of shopping
opportunities.
(B) The locations of super-malls, including the SM Mall of Asia, are carefully
chosen to limit competition between super-malls.
(C) The SM Mall of Asia is home to the only Olympic-sized ice skating rink in
the Phillipines.
(D) The opening of a new super-mall has increased the number of weekend
tourists visiting Manila, and these tourists typically visit only one super-mall.
(E) The success of a super-mall is often dependent on the major retailers
chosen as anchor stores, and the SM Mall of Asia has more anchor stores
than other Manila super-malls.
IMO: E
OA: D
CR Type: Paradox
Style:
My take: SM mall = threatening other super mall and has + effect on small
business
POE:
A. Opportunity doesnt resolve the issue at hand.
B. Doesnt explain why there is still a competition.
C. So how does it compete with super mall?
D. Correct one Choice (D) gives us a reason why all businesses might
benefit- an increased number of weekend tourists- and a reason why
other super-malls might not-that the tourists only visit one super-mall.
This would explain the difference
E. It state about the competition, but say nothing about the + effect on
small business

22. At Allied Consulting, an employee receives a raise at the end of her first
year with the company if she has reached a certain sales target and has
received satisfactory reviews from two managers. Even though being
assigned to a rural region makes the sales target more difficult to reach, a
higher percent of first-year employees assigned to such regions receive
raises than first-year employees assigned to non-rural regions.
Which of the following, if true, most helps to explain the apparent
discrepancy?
(A) Managers are aware of the challenges involved in reaching sales targets
in rural regions, and are more likely to give satisfactory reviews to first-year
employees assigned to such regions.
(B) The average first-year employee at Allied Consulting reaches higher sales
figures than the average first-year employee at Allieds competition.
(C) Once an employee has stayed with the company for six months; she
typically has more contacts in the industry, making it easier for her to reach
her sales target.
(D) Managers at Allied Consulting give satisfactory reviews to the majority of
first-year employees.
(E) Employees assigned to rural regions typically make more sales per
customer than employees assigned to non-rural regions.
OA: A
CR Type: Paradox
Style:
My Take: To get hike an emp. must have reached a certain sales target + a
satisfactory reviews from 2 managers. But there was a salary hike of an emp.
whose sales target was more difficult.
POE:
A. Correct
B. Nothing is mention to resolve the issue; just higher sales figures are
reached.
C. Sales target achieved is mentioned no use of this.
D. Results are biased as managers give satisfactory reviews to almost all.
E. More sales per customer is mentioned but still the other part is not
reasoned

23. A study separated all of the Saradian firms in a certain industry into two
groups. The first group consisted of firms that have the largest proportion of
clients outside of Saradia, and the second consists of those with the smallest
proportion of such clients. The first group showed a much larger increase in
sales over the five-year span in which in the firms were studied. The effect
was attributed to the weakness of the Sarade, the currency of Saradia.
Which of the following, if true, would best help explain how the weakness of
the Sarade might result in the observed effect?
(A) Each year in the five-year span, the Sarade got weaker relative to the
Euro.
(B) When a countrys currency is weak, goods sold by firms in that country
are relatively inexpensive to customers who reside in countries with stronger
currencies.
(C) The weakness of the Sarade caused Saradia.s national bank to raise
interest rates, making it more expensive for Saradian firms to raise money to
fund expansion.
(D) The Sarade is the main currency of the region, so fluctuations in
exchange rates affect several neighboring countries in addition to Saradia.
(E) The firms involved in the study were not notified that the study was
taking place until after the five-year span was complete.
OA: B
CR Type: Paradox
Style

24. Regulations in Guravia dictate that commercial aircraft are subject to


government inspection and maintenance at least once per week. A proposed
law would allow airlines to opt out
of the required inspection and maintenance. Opponents object that the
airlines may attempt to cut costs and perform inadequate safety procedures.
But since airlines are accountable to their customers, and air travelers
consistently rate safety their first priority in choosing an airline, airlines
would follow safety procedures at least as thorough as those currently
undertaken by government inspectors and maintenance workers.
Which of the following would it be most useful to establish in order to
evaluate the argument?
(A) Whether shifting the safety procedures to the airlines themselves would
reduce the cost and time required for sufficient inspection and maintenance
(B) Whether safety regulations in other countries grant airlines more
responsibility in keeping their aircraft at or above a certain standard of
airworthiness
(C) Whether government inspectors and maintenance workers would be
hired by the airlines to perform tasks similar to their current assignments
(D) Whether airlines that are not known for their safety records would suffer
financially as a result of the proposed law
(E) Whether airline customers would have a way to determine which airlines
followed safety procedures at or above the standard established by
government inspectors and maintenance workers
CR Type: Evaluate.
Time: 3:15
OA: IMO: E
Style:
My take: Argument: Airlines would follow safety procedures at least as
thorough as those currently undertaken by government inspectors and
maintenance workers.
A.l follow s.pro. >= gov insp. + Maint. Workers.
POE:
A. Cost is not important here. Out
B. Safety regulation in other countries granting more responsibility wont
mind the author -- out.

C. Hiring the workers is also of no importance. -- out


D. Again financial effect is not important. out
E. Yup.

25.
Frobnias economy relies on two major industries. The oil fields in south
Frobnia allow the country to export petroleum and the natural landmarks in
the north attract hundreds of thousands of tourists each year. Tourism
employs more than one-third of working Frobnians, and the oil fields employ
less than half as many. However, about half of the population of Frobnia lives
in the south.
The information given, if accurate, most strongly supports which of the
following?
(A) To remain fully staffed, tourism-related companies in Frobnia must recruit
in the south.
(B) Most of the people in Frobnia who are not employed live in the south.
(C) Many employed Frobnians in the south work for petroleum-related
companies apart from the oil fields, such as refineries.
(D) A higher proportion of Frobnians working in the north work in one of
Frobnias two major industries than the proportion of Frobnians in the south
who do.
(E) Frobnias oil fields are not a major tourist attraction, but a small number
of visitors include the oil fields as part of their travel.
CR Type: Inference
OA: IMO: D
Time: 4:15
My Take: Fro. Econo. 2 industries Oil (south)
- Tourism (North)
Tourisms Industry employs >= 1/3 of working of frobnians
population
Oil Industry employs < (of what Tourisms Industry
employs) < 1/6 of working of frobnians population.
But still half of population of Frobnia lives in south. Just a
trap to get confused.
POE:
A. Choice (A) assumes too much. There may well be enough people in
the north to staff
tourism-related companies. Beyond that,

B.

C.

D.

E.

even if it werent true, theres no reason the outside recruiting


would have to take place in the south.
B is only true if oil and tourism are the only two industries. Just
because they are the largest doesnt mean they are the only ones;
they could actually represent a small portion of the economy.
Keep in mind.
(C) explains how more people in the south might be related to the
oil industry, but theres no support for this. We dont know that
refineries (or other oil-related companies) are in the south or even
in Frobnia at all.
We know that more than one-third of working Frobnians work in the
tourism industry (which is in the north), while less than one-sixth
work in the oil fields (in the south). Thus, a higher proportion of
Frobnians work for either tourism or oil in the north than for tourism
or oil in the south. Correct one
(E) is a huge leap, and out of the scope of the passage. We have no
idea whether tourists also visit the south. The passage is concerned
with employment numbers.

26.
Among firms that transport their goods via truck, some hire drivers directly,
whereas others contract transportation to outside firms. Firms that transport
food items are twice as likely as other firms to hire drivers directly. Firms that
transport perishable food items, which must be delivered in a timely manner,
are more than five times more likely than other firms to hire drivers directly,
while firms that transport non-perishable food items are only slightly more
likely than other firms to do so.
The information above provides the most support for which of the following
hypotheses?
(A) A firms decision to hire drivers directly depends in part on whether goods
must be delivered in a timely manner.
(B) It is usually more expensive to hire drivers directly than to contract
transportation to outside firms.
(C) Firms that hire drivers directly are unlikely to do business with firms that
contract transportation to outside firms.
(D) Firms that provide truck transportation usually carry non-food items.
(E) Between one-fifth and one-half of firms that transport their goods via
truck contract transportation to outside firms.
CR Type: Inference
OA: IMO: A
Time: 4:31
Style: Evidence Gathering
My Take: The argument takes more about hiring Drivers directly and the
hiring of driver increased if the delivery time was important.

POE:
A. Choice (A) is a logical conclusion. The only difference between firms
that sell perishable food items and non-perishable food items is that
the former must have their items .delivered in a timely manner. If
such firms are so much more likely to hire drivers directly, the
timeliness required seems to have at least something to do with the
decision.
B. Choice (B) is not based on the information given, since there is
nothing in the passage about costs.
C. (C) is irrelevant, as the customers or suppliers of the businesses
mentioned do not arise in the discussion.
D. (D) may be tempting, but all we know is that more firms that transport
food items opt to hire their own drivers. If 90% of transported goods
are food items, a small percentage of that would still be greater than
all of the non-food items put together.
My Take: Consider Total delivery items 100 of which, 50 is food items
and 50 is non food items. But argument never talks about the firms
trading food items or anything else. Out
E. (E) is much too premise to be supported by a passage with no firm
numbers.
27.
Which of the following most logically completes the argument?
A common approach to algae cultivation involves covering a pond with a
greenhouse, instead of leaving the pond uncovered. Because there is a
practical limit on the size of a greenhouse, this method limits the extent of
the algae-growing system. Nevertheless, covering the pond results in a much
higher algae yield because.
.
(A) The greenhouse prevents contamination from invasive bacteria, which
can destroy entire species of algae.
(B) The greenhouse changes the quality of sunlight that reaches the algae,
causing more frequent genetic mutations.
(C) Leaving the pond uncovered is an acceptable solution depending on the
species of algae being cultivated.
(D) Covered ponds better mimic the artificial environment of a
photobioreactor, which is the most effective tool to preserve certain
threatened types of algae.
(E) The limited available space makes it more likely that only one species will
be cultivated in a given pond.
CR Type: Inference
OA: IMO: A

Time: 2:36
Style: Evidence gathering
My Take: Algae Cultivation 1. Covered pond with Greenhouse
2. Uncovered pond
Point 1 -> limit on the extent of the algae- growing system.
But, Point 1 -> higher algae yield
May be covered pond is protecting algae from something.
POE:
A. Ok
B. Genetic mutations dont cause the increase in yield.
C. Argument doesnt talk about the classification of algae for its
cultivation.
D. We are not concern with threatened types of algae
E. Space doesnt make more likely or less likely and even on that it talks
about only one species.

28.
Which of the following most logically completes the passage?
Both silvicultural clearcutting and commercial clearcutting involve removing
a high percentage of trees from a forested area. Silvicultural clearcutting
involves removing nearly every tree from the area in order to create an
environment suitable for a species to regenerate after the trees are
removed. However, commercial clearcutting is usually not as extensive,
because.
.
(A) removing all the trees from an area does not always create an
environment in which a species can regenerate.
(B) other forested areas are subject to the silvicultural method, ensuring that
species will regenerate in other locations.
(C) commercial clearcutting typically takes place in much larger areas, and is
more common in developing countries.
(D) forested areas that are commercially clearcut are not home to species
that are as environmentally important as those in areas that are
silviculturally clearcut.
(E) the goal of commercial clearcutting is to remove only commercially
valuable trees, and not every tree fits that description.
CR Type: Inference
OA: IMO: E

Time: 2:15.
Style: Evidence Gathering
My Take: SilviCut. And comm. cut -> high % of removal of trees.
S.C cut = every tree cut down for species
C.C cut = is not extensive?
POE:
A. It is not extensive, so a is incorrect.
B. Species will regenerate no where the intension of the argument is
describe as in this.
C. This is narrow very narrow specific to developing countries, which we
have no idea.
D. The importance to species is provided, but this is not specified
anywhere in the argument.
E. This is better.

29.
Which of the following most logically completes the passage?
For the past several years, cellular phone service has not been available in
Volsinia, despite service being available in neighboring Rolisica. Recently two
companies, Cyberdyne and
Noratech, have each established the infrastructure necessary to provide
cellular service to Volsinia residents. Although the services offered by the
companies are similarly priced and are equally easy to use, Cyberdynes
product is likely to dominate the market,
Because.
.
(A) Cyberdyne has a much larger share of the international cellular service
market than Noratech does.
(B) the number of Volsinia residents who will purchase cellular service is
likely to double over the next few years.
(C) Noratechs infrastructure is better designed to accommodate population
growth in Volsinia.
(D) both Cyberdyne and Noratech sell mobile phones in addition to cellular
service.

(E) Cyberdyne is the only cellular service provider with a substantial


presence in Rolisica, and many Volsinia residents frequently travel between
the two countries
CR Type: Inference
OA: IMO: E
Time: 1:27
Style:
My take: cyberdyne product is likely to dominate the market.
POE:
A.
B.
C.
D.
E.

Share is not the deciding factor.


Doubling residents dont indicate the preference.
Then why is Cyberdyne going to prosper.
Both sell addition service still why Cyberdyne.
Prefect.

30.
Test Code 31: Folder Path : C:\MGMAT Class torrent\Paper Tests\Gmat Paper
Tests I.zip
CR Section 6
The chanterelle, a type of wild mushroom, grows beneath host trees such as
the Douglas fir, which provide it with necessary sugars. The underground
filaments of chanterelles, which extract the sugars, in turn provide nutrients
and water for their hosts. Because of this mutually beneficial relationship,
harvesting the chanterelles growing beneath a Douglas fir seriously
endangers the tree.
Which of the following, if true, casts the most doubt on the conclusion drawn
above?
(A) The number of wild mushrooms harvested has increased in recent years.

(B) Chanterelles grow not only beneath Douglas firs but also beneath other
host trees.
(C) Many types of wild mushrooms are found only in forests and cannot
easily be grown elsewhere.
(D) The harvesting of wild mushrooms stimulates future growth of those
mushrooms.
(E) Young Douglas fir seedlings die without the nutrients and water provided
by chanterelle filaments.
CR Type : weaken
OA: IMO: D
Time:
Style: Concl: Harvesting Chanterelles growing beneath a douglas fir seriously
endangers the tree
Bcoz, of the mutually beneficial relationship
POE:
A. This would again strengthen the argument.
B. If any would strengthen,bocz the effect on Douglas would be the
same.
C. It doesnt take more about the conclusion.
D. The harvesting would actually benefit Douglas. Thus weakening the
argument.
E. This is actually strengthening the argument.

31.
The reason much refrigerated food spoils is that it ends up out of sight at the
back of the shelf. So why not have round shelves that rotate? Because such
rotating shelves would have just the same sort of drawback, since things
would fall off the shelves' edges into the rear corners.
Which of the following is presupposed in the argument against introducing
rotating shelves?

(A) Refrigerators would not be made so that their interior space is cylindrical.
(B) Refrigerators would not be made to have a window in front for easy
viewing of their contents without opening the door.
(C) The problem of spoilage of refrigerated food is not amenable to any
solution based on design changes.
(D) Refrigerators are so well designed that there are bound to be drawbacks
to any design change.
(E) Rotating shelves would be designed to rotate only while the refrigerator
door was open.
CR Type: Assumption.
OA: A; IMO: B
Time:
Style:
Stuarts Comment: Like many assumption questions, we can predict the
answer to this one: for it to be possible for food to fall off into the corners,
the author has to assume that your fridge HAS corners! Choice (a) is a great
match for this prediction.
Denial Test: Like many assumption questions, we can predict the answer to
this one: for it to be possible for food to fall off into the corners, the author
has to assume that your fridge HAS corners! Choice (a) is a great match for
this prediction.

32. It would cost Rosetown one million dollars to repair all of its roads. In the
year after completion of those repairs, however, Rosetown would thereby
avoid incurring three million dollars worth of damages, since currently
Rosetown pays that amount annually in compensation for damage done to
cars each year by its unrepaired roads.

Which of the following, if true, gives the strongest support to the argument
above?
(A) Communities bordering on Rosetown also pay compensation for damage
done to cars by their unrepaired roads.
(B) After any Rosetown road has been repaired several years will elapse
before that road begins to damage cars.
(C) Rosetown would need to raise additional taxes if it were to spend one
million dollars in one year on road repairs.
(D) The degree of damage caused to Rosetowns roads by harsh weather can
vary widely from year to year.
(E) Trucks cause much of the wear on Rosetowns roads, but owners of cars
file almost all of the claims for compensation for damage caused by
unrepaired roads.
CR Type: Strengthen
OA: B; IMO: A/E
Time:
Style:
My Take: rosetown pays the amount annually in compensation for damage
done to cars each year by its unrepaired roads.
Concl: After repairing roads, rose town would avoid incurring three million
dollars worth of damages.
POE:
A. Who is paying for the compensation for damage done to cars is not
important.
B. This is good.(from students: if the roads don't damage the cars for
years to come, the money spent on settling damages will be saved
For more: http://www.manhattangmat.com/forums/cr-it-would-costrosetown-t8981.html)
C. Again the need of additional taxes is not important.
D. This specifies the reason for road damage, which is not important.
E. Which vehicles are more damage due to unrepaired roads are not
important

33. Two experimental garden plots were each planted with the same number
of tomato plants. Magnesium salts were added to the first plot but not to the
second. The first plot produced 20 pounds of tomatoes and the second plot

produced 10 pounds. Since nothing else but water was added to either plot,
the higher yields in the first plot must been due to the magnesium salts.
Which of the following, if true, most seriously weakens the argument above?
(A) A small amount of the magnesium salts from the first plot leached into
the second plot.
(B) Tomato plants in a third experimental plot, to which a high-nitrogen
fertilizer was added, but no magnesium salts, produced 15 pounds of
tomatoes.
(C) Four different types of tomatoes were grown in equal proportions in each
of the plots.
(D)Some weeds that compete with tomatoes cannot tolerate high amounts of
magnesium salts in the soil.
(E)The two experimental plots differed from each other with respect to soil
texture and exposure to sunlight.
CR Type: Weaken
OA: IMO: E
Time:
Style: Based on ConclusionAssumption is weakened to weaken the
argument.
My take:
1st Plot
Same no of tomato plants
Magnesium salts added
20 pounds of tomatoes
Nothing else only water to both plot

2nd Plot
Same no of tomato plants
Mag. Salts not added.
10 pound of tomatoes.
Nothing else only water to both plot

Concl: Higher Yields in the first plot is due to magnesium salts.


Assumption: everything is equal before planting tomatoes.
E: Best weaken the argument.

34. Archaeologists have found wheeled ceramic toys made by the Toltec,
twelfth-century inhabitants of what is now Veracruz. Although there is no
archaeological evidence that the Toltec used wheels for anything but toys,
some anthropologists hypothesize that wheeled utility vehicles were used to
carry materials needed for the monumental structures the Toltec produced.
Which of the following, if true, would most help the anthropologists explain
the lack of evidence noted above?
(A) The Toltec sometimes incorporated into their toys representations of
utensils or other devices that served some practical purpose.
(B) Any wheeled utility vehicles used by the Toltec could have been made
entirely of wood, and unlike ceramic, wood decays rapidly in the humid
climate of Veracruz.
(C) Carvings in monument walls suggest that the Toltec's wheeled ceramic
toys sometimes had ritual uses in addition to being used by both children
and adults as decorations and playthings.
(D) Wheeled utility vehicles were used during the twelfth century in many
areas of the world, but during this time wheeled toys were not very common
in areas outside Veracruz.
(E) Some of the wheeled ceramic toys were found near the remains of
monumental structures.
CR Type: Strengthen Question
OA: IMO: B
Time:
Style: Strengthening the evidence.
My Take: some Anthro. = wheeled utility vehicles were used to carry
materials, but no evidence found to satisfy the above statement.
POE:
A. It says Toys represents utensils or other devices (Devices can be
vehicles/ cant be vehicles).
B. This is better than A.
C. Not saying anything about the use of Wheel utility vehicles.
D. Dont show any resemblance between Utility vehicles and wheel toys.
E. No information about wheel utility vehicles.

35. Demographers doing research for an international economics newsletter


claim that the average per capita income in the country of Kuptala is
substantially lower than that in the country of Bahlton. They also claim,
however, that whereas poverty is relatively rare in Kuptala, over half the
population of Bahlton lives in extreme poverty. At least one of the
demographers' claims must, therefore, be wrong.
The argument above is most vulnerable to which of the following criticisms?
(A) It rejects an empirical claim about the average per capita incomes in the
two countries without making any attempt to discredit that claim by offering
additional economic evidence.
(B) It treats the vague term "poverty" as though it had a precise and
universally accepted meaning.
(C) It overlooks the possibility that the number of people in the two countries
who live in poverty could be the same even though the percentages of the
two populations that live in poverty differ markedly.
(D) It fails to show that wealth and poverty have the same social significance
in Kuptala as in Bahlton.
(E) It does not consider the possibility that incomes in Kuptala, unlike those
in Bahlton, might all be very close to the country's average per capita
income.
CR Type: flaw in reasoning (Inference type)
OA: E; IMO: C/E
Time:
Style: evidence based
My Take: Avg. per cap. Income

< Avg. per cap. Income


(Kuptala)

(Bahlton)
Poverty

<
(Kuptala)

(1/2) Poverty
(Bahlton)

Conc: So one of the above claim is wrong.


POE:
A. Which claim is wrong or right is not mention O.F.S
B. Never seen in the argument.
C. Assume Poverty of Kuptala = 5 ,Poverty of Bahlton = 10; Number of
people of Bahlton = 200 and Number of people of Kuptala = 200
So, though Percentage of poverty of Kuptala is different from that of
bahlton ,this doesnt state anything of per capita income

It may happen the extra people in Bahlton have more or less income
as compare to that those In Kuptala
D. Out of scope.
E.
http://www.manhattangmat.com/forums/demographers-doingresearch-for-an-international-t4554.html

36. Normally, increases in the price of a product decrease its sales except
when the price increase accompanies an improvement in the product. Wine
is unusual, however. Often increases in the price of a particular producer's
wine will result in increased sales, even when the wine itself is unchanged
Which of the following, if true, does most to explain the anomaly described
above?
(A)The retail wine market is characterized by an extremely wide range of
competing products.
(B) Many consumers make decisions about which wines to purchase on the
basis of reviews of wine published in books and periodicals.
(C) Consumers selecting wine in a store often use the price charged as their
main guide to the wine's quality.
(D) Wine retailers and producers can generally increase the sales of a
particular wine temporarily by introducing a price discount.
(E) Consumers who purchase wine regularly generally have strong opinions
about which wines they prefer.
CR Type: Discrepancy
OA: IMO: C
Time:
Style: (Addition data can be used)
My take: Incre. In price of wine -> Incre. In sales, while the wine itself
unchanged.
POE:
C. Is the best
E. Opinion not important.

*****
37. Advertisement:
The world's best coffee beans come from Colombia. The more Colombian
beans in a blend of coffee, the better the blend, and no company purchases
more Colombian beans than Kreemo Coffee, Inc. So it only stands to reason
that if you buy a can of Kreemo's coffee, you're buying the best blended
coffee available today.
The reasoning of the argument in the advertisement is flawed because it
overlooks the possibility that
(A) the equipment used by Kreemo to blend and package its coffee is no
different from that used by most other coffee producers
(B) not all of Kreemo's competitors use Colombian coffee beans in the blends
of coffee they sell
(C) Kreemo sells more coffee than does any other company
(D) Kreemo's coffee is the most expensive blended coffee available today
(E) the best unblended coffee is better than the best blended coffee
CR Type: Flaw (Inference related)
OA: C; IMO: A
Time:
Style:
POE:
The answer is C because we're looking for an answer that justifies their
large purchase of beans. Just because you purchase a large amount of
beans, it doesn't mean that the blend will contain a large quantity of
beans - could be that they just sell a lot of coffee. Thus, it may not be the
best coffee because they may be skimping on the beans.

Professor: Everybody seems to be on the right track with this one, but I
wanted to respond to a few complaints. This is a VERY realistic GMAT
question. There are no ambiguous answers. The problem is only if you
spend too much time writing stories in your head, in which case, any
answer choice can begin to look good. This argument says they buy the
most Colombian beans, but think of it this way:
If I buy 10 beans and you only buy one bean, it may look like I must have
the best coffee.

But if you only sell one cup of coffee, your one bean means the overall
quality of your coffee is super excellent.
If I then sell 10,000 cups of coffee, splitting my ten beans between them,
the overall quality of my coffee is super poor.
This is why the answer is what it is. Don't get into thought processes
involving the history of the company, why people buy their coffee, what
effect technology may have...ANYTHING that isn't in the passage itself.
Keep in mind that this question asks you weaken THE ARGUMENT, not THE
CONCLUSION, so you won't be bringing in any new information.
Student:
The statement says that ratio of Colombian beans to total beans in a blend
affects quality.
So
Colombian beans
---------------- = quality (roughly)
Total number of beans
The conclusion is therefore the more Colombian beans purchased, the higher
the quality.
However the second part of the equation is not clear, i.e. how many beans
total they use.
So if Kreemo's sales are sufficiently high, they may be diluting their coffee
with lesser quality beans, and hence not have the highest quality coffee.

38.
The only purpose for which a particular type of tape is needed is to hold
certain surgical wounds closed for ten daysthe maximum time such
wounds need tape. Newtape is a new brand of this type of tape. Newtapes
salespeople claim that Newtape will improve healing because Newtape
adheres twice as long as the currently used tape does.
Which of the following statements, if true, would most seriously call into
question the claim made by Newtapes salespeople?
(A) Most surgical wounds take about ten days to heal.
(B) Most surgical tape is purchased by hospitals and clinics rather than by
individual surgeons.
(C) The currently used tapes adhesiveness is more than sufficient to hold
wounds closed for ten days.
(D) Neither Newtape nor the currently used tape adheres well to skin that
has not been cleaned.
(E) Newtapes adhesion to skin that has been coated with a special chemical
preparation is only half as good as the currently used tapes adhesion to
such coated skin.
CR Type: Weaken
OA: C; IMO: E
Time:
Style:
POE: The claim is that it "will prove healing."
Improving healing is based on 1) how long the wound takes to heal and 2)
how long the tape holds.
The sentence says that the max healing time is 10 days.
So a) would actually improve the claim because it undermines the "max 10
days" thing, suggesting a potential use for the new tape.
b) As someone said, is out of scope.
d) and e) are referencing situations (clean skin, chemically treated skin) that
are not referenced in the opening sentence, and one can assume such
situations are irrelevant or redundant (what kind of doctor is going to dress a
dirty wound?)
c) doesn't contradict the "max 10 days" and it goes on to state that current
tape will hold for 10 days or more, meaning the new tape would act no
differently, thereby NOT improving (or deterring) healing.
So by logic AND processes of elimination it's gotta be C.

39. The recent upheaval in the office-equipment retail business, in which


many small firms have gone out of business, has been attributed to the
advent of office equipment superstores whose high sales volume keeps
their prices low. This analysis is flawed, however, since even today the
superstores control a very small share of the retail market.
Which of the following, if true, would most weaken the argument that the
analysis is flawed?
(A) Most of the larger customers for office equipment purchase under
contract directly from manufacturers and thus do not participate in the retail
market.
(B) The superstores heavy advertising of their low prices has forced prices
down throughout the retail market for office supplies.
(C) Some of the superstores that only recently opened have themselves gone
out of business.
(D) Most of the office equipment superstores are owned by large retailing
chains that also own stores selling other types of goods.
(E) The growing importance of computers in most offices has changed the
kind of office equipment retailers must stock.
CR Type: Flaw
OA: B; IMO: A
remember that you must stay within the scope of the argument. you should
develop a keen sense for what is, and what isn't, relevant to the issue(s) at
hand.
in this problem, the argument is concerned solely with the retail market; nonretail business, such as the direct contracting mentioned in choice (a), is
irrelevant to the discussion.
here's the basic skeleton of the argument:
* someone has attributed small firms' struggling in the equipment retail
business to the entry of superstores into that retail market.
* this is flawed because the superstores control a low % of the market.
your goal here is to weaken the argument, which, as always, is done by
undermining assumptions.
it's actually fairly straightforward to pick up on the key assumption in the
argument here, because a connection is made, without any explicit
justification, between (a) the superstores' low market share in the retail
business and (b) their purported inability to affect the bottom line of the
other firms in the business.

therefore, the argument (the original argument, which the narrator says is
flawed) makes the following assumption: stores with a small market share
can't possibly affect the other firms in the market.
therefore, we need to find an answer choice that gives a way in which stores
can affect the other firms in the market even if those stores have a small
market share.
choice (b) does this: the stores' heavy advertising is independent of their
small market share, so this choice gives a way in which the superstores,
despite a small market share, can have a significant impact on the rest of the
market.
to reiterate, choice (a) is irrelevant, because the argument is concerned
solely with the retail market, and so any consideration of the non-retail
market is outside the scope of the argument.
(d) is completely irrelevant.
the scope of the passage is limited to the retail market for office supplies,
particularly the impact of superstores upon that market.
the ownership of the superstores isn't relevant to this line of reasoning; there
is nothing whatsoever in the passage that has any relation to the ownership
of the stores.
furthermore, it makes no difference what other kinds of stores are owned by
the same owners, as those other stores do business outside the office supply
retail market (and are therefore outside the scope of the argument).

40.
Test Code: 55
It is well known that human tears often serve to moisten the eye, protect it
from infection, and wash away irritants; such tears are called irritant or reflex
tears. Dr. Field hypothesizes that emotional tears have a different biological
function. She suggests that by shedding tears when under emotional stress
people excrete harmful chemicals that build up in such body fluids as blood
serum during emotional stress.
Each of the following, if true, provides some support for Dr. Fields hypothesis
EXCEPT:
(A) The people most likely to cry when undergoing emotional stress are less
likely to suffer from stress-related diseases than is the population at large.
(B) If a local anesthetic is applied to the surface of the eye, irritant and reflex
tears are inhibited, but emotional tears are not.
(C) The chemical composition of tears that are induced by grit in the eye is
identical to the composition of tears induced by emotional stress.
(D) The concentration of a substance that the body produces only under
conditions of emotional stress is thirty times greater in tears than in blood
serum.
(E) Patients who suffer from a condition that prevents secretion of tears
display a slower than normal physiological recovery from emotional stress.
CR Type: Except Strengthening
OA: IMO: C
Time: 3:14
Style:
My Take: Fields hypothesis that shedding tears under emotional stress
people excrete harmful chemicals is different from that of the tears produce
due to infection or irritants.
POE:

The hypothesis is about the "emotional tears" - that they serve a different
function of excreting harmful chemicals that accumulate DURING emotional
stress.
We have 4 STRENGTHENERS and 1 which is NOT. We have to find the one
which isn't a strengthener.
A. If this is true, it means that stress related diseases are less likely to occur
to people who cry when stressed. This could support the hypothesis since it
is possible that this is due to the fact that harmful chemicals are removed
from the body in these emotional tears.
B. This serves as a differentiator between the emotional tears and the reflex
tears, which supports the hypothesis that they may have different functions
too.
C. Well if reflex tears and emotional tears have the same chemical
composition, it cannot be hypothesized that emotional tears contain harmful
chemicals excreted by the body, or even if they do this is not specific to
them as irritant/reflex tears must also contain them. This weakens the
doctor's hypothesis that emotional tears have a different function of
excreting chemicals.
D. This clearly indicates that a harmful chemical produced in the body is
present in the tears, and thus strengthens the hypothesis.
E. This indicates that patients who cannot produce the emotional tears take
more time to recover from stress related problems, and this clearly suggests
that there is a relationship between the tears and stress related problems,
which strengthens the hypothesis.

41. In theory, Papua New Guinea could be a substantial exporter of tropical


crops. In actuality, it is not. The reason is that 97 percent of all land is owned
by clans and cannot be bought or sold by individuals, and thus the kinds of
realignment of properties that would be necessary to achieve maximum
production for export have been impossible to achieve.
The answer to which of the following questions would be most relevant to
evaluating the adequacy of the explanation given above?
(A) Who owns the 3 percent of the land in Papua New Guinea that is not
owned by clans?
(B) What percentage of Papua New Guineas current production of tropical
crops is consumed within the country?
(C) How much longer is land ownership by clans expected to remain the
prevailing cultural pattern in Papua New Guinea?
(D) Which of the tropical crops currently grown in Papua New Guinea could
be exported if there were a surplus for export?
(E) How does Papua New Guineas current production capacity for tropical
crops compare with the maximum capacity that property realignment would
make possible?
CR Type: Evaluating
OA: IMO: E
Time: 2:58
POE: conclusion: Papua New Guinea could NOT be a substantial exporter of
tropical crops

Why?
Premise: (evidence) 97 percent of all land is owned by clans so realignment
of properties is required to achieve maximum production.
(A) Who owns the 3 percent of the land in Papua New Guinea that is not
owned by clans?
-- Not required to know the 3%

(B) What percentage of Papua New Guineas current production of tropical


crops is consumed within the country?
-- Lets assume that they consume 100%/0%. Can it say that "realignment" is not
necessary to achieve exports?

(C) How much longer is land ownership by clans expected to remain the
prevailing cultural pattern in Papua New Guinea? -- OOS
(D) Which of the tropical crops currently grown in Papua New Guinea could
be exported if there were a surplus for export?
-- Variety of corps is OOS

(E) How does Papua New Guineas current production capacity for tropical
crops compare with the maximum capacity that property realignment would
make possible?
-- Here it seeks that how "maximum capacity" relates with "property realignment".
Say they relate 100%, so such alignment would be beneficial otherwise alignment
would be completely futile.

42. Researchers have concluded from a survey of people aged 65 that


emotional well-being in adulthood is closely related to intimacy with siblings
earlier in life. Those surveyed who had never had any siblings or who said
that at college age they were emotionally distant from their siblings were
emotionally less well adjusted at 65 than were those who had been close to
at least one brother or sister.
If the researchers conclusion is accurate, it follows that
(A) some people who attended college as young adults are likely as a result
to be emotionally better off at age 65
(B) the emotional well-being of people aged 65 depends on the emotional
well-being of their siblings
(C) it is closeness to siblings rather than just having siblings that is more
relevant to peoples emotional well-being at age 65
(D) people who are emotionally well off at college age are more likely to be
emotionally well off at age 65 as well
(E) Intimacy with siblings is more important to people at college age than it
is at age 65
CR Type: Assumption
OA: IMO: C
Time: 2:36

POE:
If the researchers conclusion is accurate, it follows that
(A) giving a different theory
(B) emotional being of old depends on emotional being of siblings - no
(C) correct
(D) doesnt fit into the argument anywhere
(E) talks about when intimacy it is important

43.
Which of the following, if true, most seriously weakens the researchers argument?
(A) As they get older, many people think more about their mortality and thus must confront
feelings of loneliness and isolation.
(B) People suffering from the emotional distress of maladjustment usually remember being less
intimate with other people than they actually were.
(C) Memory of ones past plays a greater role in the emotional well-being of older people than it
does in that of younger people.
(D) Few people can correctly identify the true sources of their emotional well-being or of their
emotional difficulties.
(E) Siblings are more likely to have major arguments and deep differences of opinion at college
age than at any other time of their lives
CR Type: Strengthen
OA: B; IMO: D
Time:

POE For correct one


(A) there is no connection to the sibling intimacy
(B) - reverses the cause and effect hence definitely weakens the argument.
(C) absurd
(D) - irrelevant to the argument. The researchers have to figure out not the persons
themselves.

44.
Test Code: 52: GMAT Section 3
In the first half of this year, from January to June, about three million
videocassette recorders were sold. This number is only 35 percent of the
total number of videocassette recorders sold last year. Therefore, total sales
of videocassette recorders will almost certainly be lower for this year than
they were for last year.
Which of the following, if true, most seriously weakens the conclusion above?
(A) The total number of videocassette recorders sold last year was lower
than the total number sold in the year before that.
(B) Most people who are interested in owning a videocassette recorder have
already purchased one.
(C) Videocassettes recorders are less expensive this year than they were
last year.
(D) Of the videocassette recorders sold last year, almost 60 percent were
sold in January.
(E) Typically, over 70 percent of the sales of videocassette recorders made in
a year occur in the months of November and December
CR Type: Weaken

OA: IMO: E
Time: 1:42
Style: Conclusion and some calculation part.
My Take: Jan Jun (this yr) No of videocassette recorders sold =~ 35% of No
of videocassette recorders sold (last yr)
Concl: Total sales of videocassette recorders (this year) = lower than last
year sales.
POE:
A. The total number of videocassette recorders sold last year was
lower than the total number sold in the year before that. (The sales
of past to past is not important).
B. Most people who are interested in owning a videocassette recorder
have already purchased one. (This will strengthen the argumentbe
careful dont think too much such as more ppl will be added or etc.)
C. Videocassettes recorders are less expensive this year than they
were last year. (Even if it is less expensive, why sales will increase?)
D. Of the videocassette recorders sold last year, almost 60 percent were
sold in January.
(This would support the argument.)
E. This would weaken the argument

45. Mud from a lake on an uninhabited wooded island in northern Lake


Superior contains toxic chemicals, including toxaphene, a banned pesticide
for cotton that previously was manufactured and used, not in nearby regions
of Canada or the northern United States, but in the southern United States.
No dumping has occurred on the island. The island lake is sufficiently
elevated that water from Lake Superior does not reach it.
The statements above, if true, most strongly support which of the following
hypothesis?
(A) The waters of the island lake are more severely polluted than those of
Lake Superior.
(B) The toxaphene was carried to the island in the atmosphere by winds.
(C) Banning chemicals such as toxaphene does not aid the natural
environment.
(D) Toxaphene has adverse effects on human beings but not on other
organisms.

(E) Concentrations of toxaphene in the soil of cotton-growing regions are not


sufficient to be measurable.
CR Type: Inference
OA: B; IMO: A
Time: 3:03
Style: Logical (Read it properly)
My Take: Mud of lake on island near Lake sup = Toxic; No dumping on island.
Island lake = elevated so water from lake sup doesnt reach Island lake.
POE:
A. The waters of the island lake are more severely polluted than those
of Lake Superior.
(Does being toxic mean it is polluted/ the pollution of Lake Superior is
unknown).
B. The toxaphene was carried to the island in the atmosphere by winds.
(Nice one)
C. Banning chemicals such as toxaphene does not aid the natural
environment. (This is not a kind of weakening argument; it is Inference
so this is outside information).
D. Toxaphene has adverse effects on human beings but not on other
organisms. (Effects on human or organism is not in question).
E. Concentrations of toxaphene in the soil of cotton-growing regions
are not sufficient to be measurable (Concentration is not important).

46. Last year in the United States, women who ran for state and national
offices were about as likely to win as men. However, only about fifteen
percent of the candidates for these offices were women. Therefore, the
reason there are so few women who win elections for these offices is not that
women have difficulty winning elections but that so few women want to run.
Which of the following, if true, most seriously undermines the conclusion
given?
(A) Last year the proportion of women incumbents who won reelection was
smaller than the proportion of men incumbents who won reelection.

(B) Few women who run for state and national offices run against other
women.
(C) Most women who have no strong desire to be politicians never run for
state and national offices.
(D) The proportion of people holding local offices who are women is smaller
than the proportion of people holding state and national offices who are
women.
(E) Many more women than men who want to run for state and national
offices do not because they cannot get adequate funding for their campaigns
CR Type: Weaken
OA: E; IMO: B
Time: 5:30
My Take: Last yr: Women were as = Men to win election to ran offices, but
only 15% of the candidates of offices = women (Author assumption: 50% of
candidates should be women)
Concl: women lost election due to difficulty in winning election and not bcoz
few women want to run.
POE:
A. Last year the proportion of women incumbents (Occupant) who won
reelection was smaller than the proportion of men incumbents who
won reelection. (Wining Reelection is not important)
B. Few women who run for state and national offices run against other
women. ( even if we see few women that means out of 50 less than
25% ,consider 25 % fail but what about rest 10% ?)
C. Most women who have no strong desire to be politicians never
run for state and national offices. (Strengthen the argument).
D. The proportion of people holding local offices who are women is
smaller than the proportion of people holding state and
national offices who are women. (Out of scope)
E. Many more women than men who want to run for state and national
offices do not because they cannot get adequate funding for their
campaigns (Yup correct one)

47. Samples from a ceramic vase found at a tomb in Sicily prove that the
vase was manufactured in Greece. Since the occupant of the tomb died
during the reign of a Sicilian ruler who lived 2,700 years ago, the location of
the vase indicates that there was trade between Sicily and Greece 2,700
years ago.
Which of the following is an assumption on which the argument depends?

(A) Sicilian potters who lived during the reign of the ruler did not produce
work of the same level of quality as did Greek potters.
(B) Sicilian clay that was used in the manufacture of pottery during the
rulers reign bore resemblance to Greek clay used to manufacture pottery at
that time.
(C) At the time that the occupant of the tomb was alive, there were ships
capable of transporting large quantities of manufactured goods between
Sicily and Greece.
(D) The vase that was found at the Sicilian tomb was not placed there many
generations later by descendants of the occupant of the tomb.
(E) The occupant of the tomb was not a member of the royal family to which
the Sicilian ruler belonged.
CR Type: Assumption
OA: IMO: D
Time: 3:56
Style: data gathering
My take: Ceramic vase manf. In Greece found in Sicily and as all occupant
died
Concl: Trade between sicily and Greece exist
Assumption: No on after 2700 brought the vase bcoz even if trade may have
existed after 2700 the argument ask for trade started since 2700 yrs ago.
POE:
A. Sicilian potters who lived during the reign of the ruler did not
produce work of the same level of quality as did Greek potters. (This
means they were different, but how does this claims that the trade
exist due to this different).
B. Sicilian clay that was used in the manufacture of pottery during the
rulers reign bore resemblance to Greek clay used to manufacture
pottery at that time.(The Clay is not of important)
C. At the time that the occupant of the tomb was alive, there were ships
capable of transporting large quantities of manufactured goods
between Sicily and Greece.(Even if the ships were capable, this
doesnt mean that trade exist between Sicily and greece)
D. The vase that was found at the Sicilian tomb was not placed there
many generations later by descendants of the occupant of the tomb.
---Correct
E. The occupant of the tomb was not a member of the royal family to
which the Sicilian ruler belonged. out of scope
48. In several cities, the government is going ahead with ambitious
construction projects despite the high office-vacancy rates in those cities.

The vacant offices, though available for leasing, unfortunately do not meet
the requirements for the facilities needed, such as court houses and
laboratories. The government, therefore, is not guilty of any fiscal
wastefulness.
Which of the following is an assumption on which the argument above
depends?
(A) Adaptation of vacant office space to meet the governments
requirements, if possible, would not make leasing such office space a more
cost-effective alternative to new construction.
(B) The government prefers leasing facilities to owning them in cases where
the two alternatives are equally cost-effective.
(C) If facilities available for leasing come very close to meeting the
governments requirements for facilities the government needs, the
government can relax its own requirements slightly and consider those
facilities in compliance.
(D) The governments construction projects would not, on being completed,
add to the stock of facilities available for leasing in the cities concerned.
(E) Before embarking on any major construction project, the government is
required by law to establish beyond any reasonable doubt that there are no
alternatives that are more cost-effective.
CR Type: Assumption
OA: A; IMO: D
Time: 3:05
My take: Gov = Amb. Const. projects despite the high office vacancy rate.
Vacant office is used for lease. But requirement not met
Concl: Gov not guilty of any fiscal wastefulness/
POE:
A.
B.
C.
D.
E.

Correct
This will undermine the conclusion.
This will undermine the conclusion.
Little relevant
The government has a valid reason as the buildings are not suitable for
courts.

49. Potato cyst nematodes are a pest of potato crops. The nematodes can lie
dormant for several years in their cysts, which are protective capsules, and
do not emerge except in the presence of chemicals emitted by potato roots.
A company that has identified the relevant chemicals is planning to market
them to potato farmers to spread on their fields when no potatoes are
planted; any nematodes that emerge will soon starve to death.
Which of the following, if true, best supports the claim that the companys
plan will be successful?
(A) Nematodes that have emerged from their cysts can be killed by ordinary
pesticides.
(B) The only part of a potato plant that a nematode eats is the roots.
(C) Some bacteria commonly present in the roots of potatoes digest the
chemicals that cause the nematodes to emerge from their cysts.
(D) Trials have shown that spreading even minute quantities of the chemicals
on potato fields caused nine-tenths of the nematodes present to emerge
from their cysts.
(E) The chemicals that cause the nematodes to emerge from their cysts are
not emitted all the time the potato plant is growing.
CR Type: Strengthen
OA: IMO: D
Time: > 3
My take: Nematodes (Pest) = emerge only when chemicals emitted by potato
roots.
Comp claim: Relevant chemicals = spread on farmer -> any nematodes that
emerge will die due to starve.
POE:
A. Nematodes that have emerged from their cysts can be killed by
ordinary pesticides. (
Ordinary pesticides are of no importance.)
B. The only part of a potato plant that a nematode eats is the roots.
(Nowhere is it mentioned that nematode eats the roots.)
C. Some bacteria commonly present in the roots of potatoes digest the
chemicals that cause the nematodes to emerge from their cysts. (Not
important.)
D. Trials have shown that spreading even minute quantities of the
chemicals on potato fields caused nine-tenths of the nematodes
present to emerge from their cysts. Correct

E. The chemicals that cause the nematodes to emerge form their cysts
are not emitted all the time the potato plant is growing. (It is not
important.)

50. In malaria-infested areas, many children tend to suffer several bouts of


malaria before becoming immune to the disease. Clearly, what must be
happening is that those childrens immune systems are only weakly
stimulated by any single exposure to the malaria parasite and need to be
challenged several times to produce an effective immune response.
Which of the following, if true, most seriously undermines the explanatory
hypothesis?
(A) Immediately after a child has suffered a bout of malaria, the childs
caregivers tend to go to great lengths in taking precautions to prevent
another infection, but this level of attention is not sustained.
(B) Malaria is spread from person to person by mosquitoes, and mosquitoes
have become increasingly resistant to the pesticides used to control them.
(C) A certain gene, if inherited by children from only one of their parents, can
render those children largely immune to infection with malaria.
(D) Antimalaria vaccines, of which several are in development, are all
designed to work by stimulating the bodys immune system.
(E) There are several distinct strains of malaria, and the bodys immune
response to any one of them does not protect it against the others
CR Type: Weaken
OA: E; IMO: C
Time: 4:00
My take:
POE:
A. This talks about attention required after malaria, doesn't help in
undermining the given explanation.
B. The argument talks of immunity towards the parasights and has
nothing to do with mosquitoes.
C. This doesn't explain why immune system doesn't work in case of
malaria.
D. The explanation talks nothing about the substances outside the human
body so this is out of the scope.

E. This is perfect as it tells that not all types of malaria are opposed by
the immune system of the body.

***
51. An advertisement designed to convince readers of the great durability of
automobiles manufactured by the Deluxe Motor Car Company cites as
evidence the fact that over half of all automobiles built by the company since
1970 are still on the road today, compared to no more than a third for any
other manufacturer.
Which of the following, if true, most strongly supports the advertisements
argument?
(A) After taking inflation into account, a new Deluxe automobile costs only
slightly more than a new model did in 1970.
(B) The number of automobiles built by Deluxe each year has not increased
sharply since 1970.
(C) Owners of Deluxe automobiles typically keep their cars well maintained.
(D) Since 1970, Deluxe has made fewer changes in the automobiles it
manufactures than other car companies have made in their automobiles.
(E) Deluxe automobiles have been selling at relatively stable prices in recent
years.
CR Type: Strengthen
OA: B; IMO: C
Time: 2:21
From Students:
Clearly A,C,E are irrelevant or will weaken the argument
B says The number of automobiles built by Deluxe each year has not
increased sharply since 1970.
Consider the opposite
The number of automobiles built by Deluxe each year has increased sharply
since 1970. The argument points out that the durability of the cars is based
on the longevity of the car. If they made only 1000 cars from 70-2000 and

200K in 2001-2005 and 100K cars are on the market, Is that durability
caused by longevity? No.
Hence this is a good choice, even though this is more like an assumption.
Assumptions are nothing but unstated premises that will strengthen the
conclusion or needed for the conclusion to be valid, because without the
assumption the conclusion will fall apart.
D is saying that few changes have been done by Deluxe. sort of supports the
argument. But not entirely because the durability in the argument is based
on the longevity and not how many changes. Just because Deluxe chose not
to change much over a long time does not mean that lack of changes (not
lack of repairs/recalls) for a long time means longevity.

52. Advertisement:
Of the many over-the-counter medications marketed for the relief of sinus
headache. SineEase costs the least per dose. And SineEase is as effective
per dose as the most effective of those other medications. So for relief from
sinus headaches, SineEase is the best buy.
Which of the following, if true, most seriously weakens the argument above?
(A) Most of the over-the-counter medications marketed for the relief of sinus
headache are equally effective per dose in providing such relief.
(B) Many of the over-the-counter medications marketed for the relief of sinus
headache contain the same active ingredient as SineEase.
(C) People who suffer from frequent sinus headaches are strongly advised to
consult a doctor before taking any over-the-counter medication.
(D) An over-the-counter medication that is marketed for the relief of
symptoms of head cold is identical in composition to SineEase but costs less
per dose.
(E) The per dose price for any given over-the-counter medication marketed
for the relief of sinus headache is higher for smaller packages than it is for
larger packages.
CR Type: Weakens
OA: D; IMO: E
Type: Conclusion verifying.
My Take: S.E/dose < other medications; S.E/dose = other medication.
Concl: S.E = best buy

POE:
A.
B.
C.
D.
E.

This is weakening the evidence which is wrong.


If same ingredient then why is S.E the best.
Doctor can suggest patients to buy S.E
Correct one.
Out of scope.

53.
In the United States, vacationers account for more than half of all visitors to
what are technically called pure aquariums but for fewer than one quarter
of all visitors to zoos, which usually include a zoo aquarium of relatively
modest scope.
Which of the following, if true, most helps to account for the difference
described above between visitors to zoos and visitors to pure aquariums?
(A) In cities that have both a zoo and a pure aquarium, local residents are
twice as likely to visit the aquarium as they are to visit the zoo.
(B) Virtually all large metropolitan areas have zoos, whereas only a few large
metropolitan areas have pure aquariums.
(C) Over the last ten years, newly constructed pure aquariums have
outnumbered newly established zoos by a factor of two to one.
(D) People who visit a zoo in a given year are two times more likely to visit a
pure aquarium that year than are people who do not visit a zoo.
(E) The zoo aquariums of zoos that are in the same city as a pure aquarium
tend to be smaller than the aquariums of zoos that have no pure aquarium
nearby.
yeah, this one is definitely (b).
a couple of comments:
* the actual numbers of visitors to zoos and pure aquariums are completely
irrelevant.
a few of the posters above seem to think that this problem somehow says the
aquariums have more visitors than do the zoos. that's not at all true.

look back at the problem: it's solely concerned with the fraction of zoo visitors who
are on vacation - i.e., from out of town - vs. the fraction of aquarium visitors who are
on vacation (i.e., from out of town).
therefore:
* we're looking for a REASON WHY AQUARIUM VISITORS ARE MORE LIKELY
TO BE FROM OUT-OF-TOWN.
after all, this is the ONLY relevant difference here.
* the only answer choice that even comes close to addressing this issue is (b). if the
aquariums are located in only a few areas, then they will only be accessible to a few
"locals"; the rest of the visitors will have to travel to go to them. by contrast, if most
areas have zoos, then zoo visitors will have little need to travel; they can just go to
their local zoo.
* none of the other choices are relevant to the distinction between vacationers
(from out of town) and non-vacationers.
when i originally read the problem statement (before i'd read the answer choices), i
thought the answer was going to say "pure aquariums are generally located in
resort areas". that answer choice isn't there, but it would also provide a very good
reason for most pure-aquarium visitors to be from out of town

54.
300 CR
Anthropologist: Violence is an extreme form of aggression, and is distinct
from the self-expression sufficient for survival under normal conditions.
Human beings in certain situations react to unpleasant stimuli with violence
but only because they are conditioned by their culture to react in this
manner.
Each of the following can be logically inferred from the anthropologists
statements EXCEPT:
(A) Not all aggression is violent.
(B) The self-expression required for survival is generally nonaggressive.
(C) Some behaviors are influenced by the cultures in which human beings
live.
(D) In normal circumstance, human beings can survive by responding
nonviolently.
(E) Violent behavior is a product of ones cultural environment
CR Type: Inference Except.
OA: IMO: B
Time: 3:25
My Take: Viol. = ext form of aggr = self exp which is suff for survival in norm.
cond.

H.B in cert suit = violence ,bcoz they are conditioned by their culture to
react.
POE:
A. Not all aggression is violent. (This is an inference; there can be many
form of aggression, violent is just a part.)
B. The self-expression required for survival is generally nonaggressive.
(Author says that Self expression sufficient for survival under
normal conditions so normal condition is important here: So this
cannot be the inference.)
C. Some behaviors are influenced by the cultures in which human beings
live. (This is the inference, culture influenced Human beings have such
behavior.: This is the inference)
D. In normal circumstance, human beings can survive by responding
nonviolently. (As Self-expression is distinct from Violence: This is the
inference.)
E. Violent behavior is a product of ones cultural environment (The
argument last statement is the same.)

55.
Philosopher: The rational pursuit of happiness is quite different from always
doing what one most strongly desires to do. This is because the rational
pursuit of happiness must include consideration of long-term consequences,
whereas our desires are usually focused on the short term. Moreover, desires
are sometimes compulsions, and while ordinary desires result in at least
momentary happiness when their goals are attained, compulsions strongly
drive a person to pursue goals that offer no happiness even when reached.
If all of the philosopher's statements are true, each of the following could be
true EXCEPT:
(A) the majority of people do not have compulsions.
(B) Attaining the goal of any desire results in momentary happiness.
(C) Most people do not pursue happiness rationally.
(D) Most people want more than their own personal happiness.
(E) All actions have long-term consequences

CR Type: Except Inference.


OA: B
A the majority of people do not have compulsions.
"doing what one most strongly desires to do.......desires are sometimes
compulsions"
B Attaining the goal of any desire results in momentary happiness.
"ordinary desires result in at least momentary happiness...
In the argument it is stated that desire are sometimes compulsions.Replace
desires with compulsions.
Attaining the goal of any compulsions results in momentary happiness.Is it
true?NO
Because the conclusion specifies compulsions strongly drive a person to
pursue goals that offer no happiness even when reached.
C Most people do not pursue happiness rationally.
"The rational pursuit of happiness is quite different from always doing what
one most strongly desires to do"
D Most people want more than their own personal happiness.
"ordinary desires result in at least momentary happiness when their goals
are attained, compulsions strongly drive a person to pursue goals that offer
no happiness even when reached"
E All actions have long-term consequences.
In the argument it is stated that" rational pursuit of happiness must include
consideration of long-term consequences,
whereas our desires are usually focused on the short term"
The philosopher believes that all actions have long term consequences but
our is based on short term.
This is true.

56.
Anger in response to insults is unreasonable, for insults are merely assertions
that someone has undesirable characteristics. If such an assertion is false,
the insulted party ought to pity the ignorance prompting the insult. If it is
true, the insulted party should be grateful for such useful information.
Which one of the following, if assumed, enables the arguments conclusion to
be properly drawn?
A. Actions prompted by ignorance do not warrant hostile reactions.
B. Anger is an unreasonable response to useful information.
C. Anger is an unreasonable response to any action that should prompt pity
or gratitude.
D. Gratitude and pity are reasonable responses to some forms of hostile or
insensitive behavior.
E. Pity is the only reasonable reaction to people with undesirable
characteristics.

CR Type: Assumption
OA: IMO: C
Time: 3:20
My Take: Anger in resp. to insult not reasonable. Insult (assertion) = someone
undesirable chara.
If asseration = fail ->pity the ignorance prompting the insult.
If asseration = pass -> grateful
POE:
A.
B.
C.
D.
E.

Too strong.
Not always
Correct
Doesnt know whether those are reasonable response.
Too strong

57.
Fares on the city-run public buses in Greenville are subsidized by city tax
revenues, but among the beneficiaries of the low fares are many people who
commute from outside the city to jobs in Greenville. Some city councilors
argue that city taxes should be used primarily to benefit the people who pay
them, and therefore that bus fares should be raised enough to cover the cost
of the service.
Each of the following, if true, would weaken the argument advanced by the
city councilors EXCEPT:
A. Many businesses whose presence in the city is beneficial to the citys
taxpayers would relocate outside the city if public-transit fares were more
expensive.
B. By providing commuters with economic incentives to drive to work, higher
transit fares would worsen air pollution in Greenville and increase the cost of
maintaining the citys streets.

C. Increasing transit fares would disadvantage those residents of the city


whose law incomes make them exempt from city taxes, and all city
councilors agree that these residents should be able to take advantage of
city-run services.
D. Voters in the city, many of whom benefit from the low transit fares, are
strongly opposed to increasing local taxes.
E. People who work in Greenville and earn wages above the nationally
mandated minimum all pay the city wage tax of 5 percent.
CR Type: Except weaken.
OA: IMO: D
Time: 5:10
Investigate more:
http://my-gmatprep.blogspot.com/2009_02_01_archive.html
http://gmatclub.com/forum/fares-on-the-city-run-public-buses-80388.html

58.
There is relatively little room for growth in the overall carpet market, which is
tied to the size of the population. Most who purchase carpet do so only once
or twice, first in their twenties or thirties, and then perhaps again in their
fifties or sixties. Thus as the population ages, companies producing carpet
will be able to gain market share in the carpet market only through
purchasing competitors, and not through more aggressive marketing.
Which one of the following, if true, casts the most doubt on the conclusion
above?
A. Most of the major carpet producers market other floor coverings as well.

B. Most established carpet producers market several different brand names


and varieties, and there is no remaining niche in the market for new brands
to fill.
C. Two of the three mergers in the industrys last ten years led to a decline in
profits and revenues for the newly merged companies.
D. Price reductions, achieved by cost-cutting in production, by some of the
dominant firms in the carpet market are causing other producers to leave the
market altogether.
E. The carpet market is unlike most markets in that consumers are becoming
increasingly resistant to new patterns and styles
CR Type: weaken
OA: D; IMO: E
Time: 3:10
From Students: D for me as it says that Price reduction is an alternate way to
increase the market share.
The argument says that companies will be able to gain market share only by
purchasing other companies not by aggressive marketing.
'D' - Because of cost cutting other companies are leaving the market. This
means there will be bigger market share to capture for the companies that
remain in the market.
From Tutor: The conclusion is: "as the population ages, companies producing
carpet will be able to gain market share in the carpet market only through
purchasing competitors, and not through more aggressive marketing".
This conclusion is a prediction. Whenever the author advances a prediction,
one necessary assumption he makes is: whatever must happen for the
prediction to come to pass will happen. And notice how extreme the
conclusion is: the ONLY way they can gain market share is through purchase
of competitors.
So, the author is assuming there are no other ways market share can be
gained (this is what must happen in order for the prediction to happen; if
there are other ways they can gain market share, then the prediction will not
necessarily happen).
In a weaken question, we find a choice that attacks the assumption...that is
what choice D is doing.

59.
In the Centerville Botanical Gardens, all tulip trees are older than any
maples. A majority, but not all, of the gardens sycamores are older than any
of its maples. All the gardens maples are older than any of its dogwoods.
If the statements above are true, which one of the following must also be
true of trees in the Centerville Botanical Gardens?
A. Some dogwoods are as old as the youngest tulip trees.
B. Some dogwoods are as old as the youngest sycamores.
C. Some sycamores are not as older as the oldest dogwoods.
D. Some tulip trees are not as old as the oldest sycamores.
E. Some sycamores are not as old as the youngest tulip trees.
Answer: E
Explanation: We know:
tulips > maples
some sycamores > maples
maples > some sycamores
all maples > dogwoods
so we know
tulips / [some] sycamores > maples > dogwoods / [some] sycamores
Important: We have no info on the relation of tulips to sycamores or
dogwoods to sycamores.
A) This is impossible. If all tulips are older than maples and all dogwoods are
younger than maples, no dogwoods could be as old as tulip trees. Eliminate.
B) This is possible but we are looking for a must be true. The oldest dogwood
could be 1 year old and the youngest sycamore 6 years old, and we could
still fulfill the terms of the question. Eliminate.
C) Again, this is possible, but it doesn't need to be true to fulfill the terms of
the problem. Eliminate.
D) Once again, this is possible. Some tulip trees could be younger than the
oldest sycamore, we have no info as to the relation between tulips and
sycamores. But it is equally possible that all tulip trees are older than the
oldest sycamore. Eliminate.
E) This is the only one that must be true. Since the youngest tulip trees are
still older than maple trees, and there are some sycamores that are younger
than maple trees, this condition must be true according to the passage

60.
Household indebtedness, which some theorists regard as causing recession,
was high preceding the recent recession, but so was the value of assets
owned by households. Admittedly, if most of the assets were owned by quite
affluent households, and most of the debt was owed by low-income
households, high household debt levels could have been the cause of the
recession despite high asset values: low-income households might have
decreased spending in order to pay off debts while quite affluent ones might
simply have failed to increase spending. But, in fact, quite affluent people
must have owed most of the household debt, since money is not lent to
those without assets. Therefore, the real cause must lie elsewhere.
The argument is structured to lead to which one of the following conclusions?
A. High levels of household debt did not cause the recent recession.
B. Low-income households succeeded in paying off their debts despite the
recent recession.
C. Affluent people probably increased their spending levels during the recent
recession.
D. High levels of household debt have little impact on the economy.
E. When people borrowed money prior to the recent recession, they did not
use it to purchase assets.
CR Type: Inference
OA: A; IMO: E
Time: 5:00
Style:
My Take: H.H indebt -> Recession, if most assets= affluent H.H, most debt =
low-income H.H:
H.H debt levels -> Recession (despite high assest values) -- Conclusion.
But author goes on saying something and at last he say the real cause must
lie else where
So correct answer is H.H debt levels -> Recession.
Note: Argument conclusion is the real cause of recession is not H.H debt
levels but something different.

61.
Government-subsidized insurance available to homeowners makes it feasible
for anyone to build a house on a section of coastline regularly struck by
hurricanes. Each major storm causes billions of dollars worth of damage in
such coastal areas, after which owners who have insurance are able to
collect an amount of money sufficient to recoup a high percentage of their
losses.
The passage provides the most support for an argument against a
government bill proposing
A. That power companies be required to bury power lines in areas of the
coastline regularly struck by hurricanes.
B. An increase in funding of weather service programs that provides a
hurricane watch and warning system for coastal areas.
C. Renewal of federal funding for emergency life-support programs in
hurricane-stricken areas.
D. Establishment of an agency committed to managing coastal lands in
ecologically responsible ways.
E. Establishment of a contingency fund protecting owners of uninsured
houses in the coastal areas from catastrophic losses due to the hurricane
damage
CR Type: Weaken (the Government bill)
OA: IMO: E
Time: 2:50
My Take: This time I have just jogged down some key words such as
Insurance feasible anyone to build house on Coastline =hurricanes
Each major storms -> millions of damage in c.a that money recoup = high %
losses
E is one which weakens the proposal.
if gov provided the compensation to uninsured for the damage, everybody
would want to be uninsured. In that case gov has to shed more money than
does currently by subsidizing the insurance. Moreover by becoming
uninsured people can save their part of premiums. Mind that the insurance is
subsidized not sponsored.

Note: Dealing with different topic at hand. Argument takes about insurance
ppl benefits and answer E takes how insurance ppl will be un-benefitted
indirectly

62.
Jane: According to an article in this newsmagazine, childrens hand-eye
coordination suffers when they spend a great amount of time watching
television. Therefore, we must restrict the amount of time Jacqueline and
Mildred are allowed to watch television.
Alan: Rubbish! The article says that only children under three are affected in
that way. Jacqueline is ten and Mildred is eight. Therefore, we need not
restrict their television viewing.
Alans argument against Janes conclusion makes which one of the following
errors in reasoning?
A. It relies on the same source that Jane cited in support of her conclusion.
B. It confuses undermining an argument in support of a given conclusion with
showing that the conclusion itself is false.
C. It does not address the main point of Janes argument and focuses instead
on a side issue.
D. It makes an irrelevant appeal to an authority.
E. It fails to distinguish the consequences of a certain practice from the
causes of the practice.
CR Type: Error In reasoning.
OA: B; IMO: E
Time: 2:52
My Take: Jane: if Great amount T.V watching -> hand-eye coordination suffers
; Jac and mid should be restrict to watch.
Alan: Wrong Conclusion; if Great amount T.V watching + age -> hand-eye
coordination suffers;
Thus should not restrict them.
By some student: J: If A (TV) then B (bad eye).
A: If A and C (younger than 3) then B. Non C, therefore non B.
Flaw: C leads to B doesn't mean non C leads to non B. (Read the CR principle thread

again.)
(B) It confuses undermining an argument in support of a given conclusion with
showing that the conclusion itself is false.
A undermine J's argument by saying only if both A and C are true, then B is certain
to be true. However he did not show that B itself is necessarily false.
Use layman's word, he showed that the article says three years old would have bad
hand eye coordination if they watch too much TV. But he didn't show whether or not
older children would suffer from the same effect.

63.
Ecologist: The Scottish Highlands were once the site of extensive forests, but
these forests have mostly disappeared and been replaced by peat bogs. The
common view is that the Highlands' deforestation was caused by human
activity, especially agriculture. However, agriculture began in the
Highlands less than 2,000 years ago. Peat bogs, which consist of
compressed decayed vegetable matter, build up by only about one foot per
1,000 years and, throughout the Highlands, remains of trees in peat
bogs are almost all at depths great than four feet. Since climate
changes that occurred between 7,000 and 4,000 years ago favored the
development of peat bogs rather than the survival of forests, the
deforestation was more likely the result of natural processes than of human
activity.
In the ecologist's argument the two portions in boldface play which of the
following roles?
A. The first is evidence that has been used in support of a position that the
ecologist rejects; the second is a finding that the ecologist uses to counter
that evidence.
B. The first is evidence that, in light of the evidence provided in the second,
serves as grounds for the ecologist's rejection of a certain position.
C. The first is a position that the ecologist rejects; the second is evidence
that has been used in support of that position.
D. The first is a position that the ecologist rejects; the second provides
evidence in support of that rejection.
E. The first is a position for which the ecologist argues; the second provides
evidence to support that position.
CR Type: Bold Face
OA: IMO: B

Time: 5:00
My Take: Ecologist: Common view: Deforestation <= human activity.
But Agriculture began in highland < 2000 yrs ago. --- Fact
Second bold face: supports the point put by ecologist, but rejects
the common view.
Note: Dissect the argument considering each sentence whether a premise or
fact.

64.
In experiments in which certain kinds of bacteria were placed in a generous
supply of nutrients, the populations of bacteria grew rapidly and genetic
mutations occurred at random in the populations. These experiments show
that all genetic mutation is random.
Which one of the following, if true, enables the conclusion to be properly
drawn?
The question here is asking us to provide the crucial link that allows us to
draw a (pretty big) conclusion that all genetic mutation is random -- in other
words, it's looking for a "must be true" in order to ensure the conclusion is
true.
We know that the experimental bacteria mutated at random and somehow,
this means that all genetic mutation occurs at random. We're just looking for
what that "somehow" is.
A) Either all genetic mutations are random or none are random.
This one is interesting. If this is true, then because the genetic mutation
occurred at random in the experiment, all other genetic mutation must occur
at random. This is the only statement that has to be true if we consider the
evidence provided and want to ensure it supports a conclusion that must be
true.
B) The bacteria tested in the experiments were of extremely common forms.

This is out of scope. If they were unusual forms it wouldn't make the
conclusion any more or less valid. Eliminate.
C) If all genetic mutations in bacteria are random, then all genetic mutations
in every other life form are random also.
This one is also interesting, since the "all" in the statement's conclusion
alludes to the fact that we should be considering other life forms, not just
bacteria. However, the experiment does not prove that all genetic mutations
in bacteria are random, just that this experiment's genetic mutations were
random. Since the answer begins with "if" and we haven't supported that
clause by the evidence, this doesn't ensure that the conclusion is valid.
Eliminate, but this is tricky.
D) The kind of environment in which genetic mutation takes place has no
effect on the way genetic mutation occurs.
Even if this is true, there are a whole host of other factors that could affect
the way genetic mutation occurs, so we couldn't reasonably conclude that all
genetic mutation is random. Eliminate.
E) The nutrients used were the same as those that nourish the bacteria in
nature.
This is out of scope and irrelevant. Eliminate.

Each December 31 in Country Q, a tally is made of the countrys total


available coal supplies that is, the total amount of coal that has been
mined throughout the country but not consumed. In 1991 that amount was
considerably lower than it had been in 1990. Furthermore, Country Q has nor
imported or exported coal since 1970.
If the statements above are true, which one of the following must also be
true on the basis of them?
A. In Country Q, more coal was mined in 1990 than was mined in 1991.
B. In Country Q, the amount of coal consumed in 1991 was greater than the
amount of coal mined in 1991.
C. In Country Q, the amount of coal consumed in 1990 was greater than the
amount of coal consumed in 1991.
D. In Country Q, the amount of coal consumed in 1991 was greater than the
amount of coal consumed in 1990.
E. In Country Q, more coal was consumed during the first half of 1991 than
was consumed during the first half of 1990.
We need a statement where we can confirmed that the difference
(production-consumption) for 1990 is higher than that of 1991 i.e. diff for

(1990)> dif for (1991).


A says that more coal was mined in 1990 than in 1991. But if the
consumption was also more than that of 1991 then the difference could be
possibly lower than that of 1991.
C says that consumption of 1990 > consumption of 1991. But still no
production figures are given. There could be a possibility that production for
both the years was same. In that case diff (1990)< diff (1991).
D says that consumption(1991)> consumption(1990). Here again the
production figures are not provided. What if production(1991) was much
higher than that of production(1990). This would mean diff (1990)< diff
(1991).
E is totally out of scope.
Hence B is true
in other words as there was no import or export & if
consumption(1991)>production(1991) then that implies that diff of prod and
cons for (1991) is less than <0 i.e. -ve,
If diff is -ve & no imports were made then the extra coal came from the last
years left over stock i.e. diff or inventory of (1990). And if there were stocks
left unconsumed in 1990 then that means that inventory (1991)>0 i.e. +ve.
Now, inventory (1990)>0 & inventory (1991)<0. Therefore, inventory
(1990)>inventory (1991). Hence, Proved. There is no other way in which
Country Q consumed more than it produced.

Spectroscopic analysis has revealed the existence of frozen nitrogen,


methane and carbon monoxide on the surface of Pluto. Such ices have a
tendency to vaporize, producing an atmosphere. Since the proportion of any
gas in such an atmosphere depends directly on how readily the
corresponding ice vaporizes, astronomers have concluded that the
components of Plutos atmosphere are nitrogen, carbon monoxide and
methane, in order of decreasing abundance.
The astronomers argument relies on which one of the following
assumptions?
A. There is no more frozen nitrogen on the surface of Pluto than there is
either frozen carbon monoxide or methane.
B. Until space probes reach Pluto, direct analysis of the atmosphere is
impossible.
C. There is no frozen substance on the surface of Pluto that vaporizes more
readily than methane but less readily than carbon monoxide.
D. Nitrogen is found in the atmosphere of a planet only if nitrogen ice is
found on the surface of that planet.
E. A mixture of nitrogen, carbon monoxide and methane is characteristic of
the substances from which the solar system formed.

CR Type: Assumption
OA: IMO: C
Time: 2:44
The conclusion is "The components of Pluto's atmosphere are N, CO and M in
order of decreasing abundance".
Let us understand the conclusion. It simply states that when the author ranks
the available gases per their abundance, the above order is true. It does NOT
means that these are the only gases on Pluto but the above is mere
hierarchy. By ranking them the author simply RULES OUT presence of any
gas that could be between N & Co and CO & M.
Analogy: When one selects A, B, C as their preferred law schools respectively,
one just means that the only school that is preferred after A is B and after B
is C. It does NOT mean there are no schools such as E, F.... but it means there
are NO schools that can come between A & B and B & C. Here, a conclusion
can be written about this particular student that he "prefers no school less
than A and more than B".
Therefore, the answer choice 'C' is correct.

To avoid economic collapse, Russia must increase its GNP by 20%. However,
due to the structure of its economy, if the 20% threshold is reached, then a
40% increase in GNP is achievable.
Assuming that the above statements are true, which one of the following
must also be true?
(A) If ethnic strife continues in Russia, then a 20% increase in GNP will be
unattainable.
(B) If a 40% increase in Russia's GNP is impossible, its economy will collapse.
(C) If Russia's GNP increases by 40%, its economy will not collapse.
(D) If the 20% threshold is reached, then a 40% increase in GNP is achievable
and a 60% increase is probable.
(E) If Russia's economy collapses, then it will not have increased its GNP by
40%.
CR Type: Inference
OA: B; IMO: C

Time: 3:00
Explanation: Diagramming will show this seemingly difficult problem to be
simply an application of the
contrapositive rule of logic: in an if-then statement, negating the conclusion
also negates the premise. The
sentence "To avoid economic collapse, Russia must increase its GNP by 20%"
can be reworded as "if
Russia does not increase its GNP by 20%, its economy will collapse." This in
turn can be symbolized as
not20%->Collapse
Where the arrow, ->, stands for "if ..., then ....
Next, symbolize the clause "if the 20% threshold is reached, then a 40%
increase is achievable" as
20%->40%
Applying the contrapositive to this statement yields
not40%->not20%
Using the transitive property (If a = b and b = c, then a = c) to combine this
with the first symbol statement
yields
not40%->Collapse
In other words, if a 40% increase in GNP is unattainable, the economy will
collapse. This is precisely what
choice (B) states. The answer is (B)

Either restrictions must be placed on freedom of speech or certain


subversive elements in society will use it to destroy this country. Since to
allow the latter to occur is unconscionable, we must restrict freedom of
speech.
The conclusion above is unsound because
(A) subversives do not in fact want to destroy the country
(B) the author places too much importance on the freedom of speech
(C) the author fails to consider an accommodation between the two
alternatives
(D) the meaning of "freedom of speech" has not been defined
(E) subversives are a true threat to our way of life
Answer: (C)

Explanation: The arguer offers two options: either restrict freedom of speech,
or lose the country. He hopes
the reader will assume that these are the only options available. This is
unwarranted. He does not state how
the so-called "subversive elements" would destroy the country, nor for that
matter why they would want to
destroy it. There may be a third option that the author did not mention;
namely, that society may be able to
tolerate the "subversives"; it may even be improved by the diversity of
opinion they offer. The answer is (C).

On completing both the course in experimental design and the


developmental psychology course, Angela will have earned a degree in
psychology. Since experimental design, which must be completed before
taking developmental psychology, will not be offered until next term, it will
be at least two terms before Angela gets her psychology degree.
If the statements above are all true, which one of the following must also be
true?
(A) The developmental psychology course Angela needs to take requires two
terms to complete.
(B) The course in experimental design is an easier course than the course in
developmental psychology.
(C) There are no prerequisites for the course in experimental design.
(D) Anyone who earns a degree in psychology from the university Angela
attends will have completed the course in experimental design.

(E) Once Angela completes the developmental psychology course, she will
have earned a degree in psychology.
OA: IMO: E
Stuart Comments:
Either the original poster mistranscribed the problem or else mistranscribed
the OA. This is an inference question requiring skill in formal logic (for that
reason, it is unlikely that you would get a question like this on the GMAT).
The first sentence of the passage reads:
"On completing both the course in experimental design and the
developmental psychology course, Angela will have earned a degree in
psychology."
This yields the following conditoinal statement:
If she completes ED AND DP, then she will have earned her psych degree.
The second sentence tells us that ED MUST be completed before DP.
Therefore:
If someone has completed DP, then they have also completed ED.
Therefore, if she completes DP, then it is sufficient to conclude that she has
also completed ED. And if she completes both DP and ED, then it is sufficient
to conclude that she will have earned her psych degree.
Accordingly, Choice E MUST be correct.
Choice D must be wrong. It improperly reverses the initial conditional
statement (without negating).
That is, in the first conditional statement, her having earned a psych degree
is a conclusion or necessary condition. Choice D is trying to use her having
earned a psych degree as a trigger or sufficient condition.
For example, if you are a dog, then you are definitely a mammal. Does that
mean that if you are a mammal, you are definitely a dog? That is what choice
D is doing.
Folks, GMAT does NOT test these formal logic subtleties.

The advanced technology of ski boots and bindings has brought a dramatic
drop in the incidence of injuries that occur on the slopes of ski resorts: from 9
injuries per 1,000 skiers in 1950 to 3 in 1980. As a result, the remainder of
ski-related injuries, which includes all injuries occurring on the premises of a
ski resort but not on the slopes, rose from 10 percent of all ski-related
injuries in 1950 to 25 percent in 1980. The incidence of these injuries,
including accidents such as falling down steps, increases with the amount of
alcohol consumed per skier.
I. Which one of the following can be properly inferred from the passage?
(A) As the number of ski injuries that occur on the slopes decreases, the
number of injuries that occur on the premises of ski resorts increases.

(B) The amount of alcohol consumed per skier increased between 1950 and
1980.
(C) The technology of ski boots and bindings affects the incidence of each
type of ski-related injury.
(D) If the technology of ski boots and bindings continues to advance, the
incidence of ski-related injuries will continue to decline.
(E) Injuries that occurred on the slopes of ski resorts made up a smaller
percentage of ski-related injuries in 1980 than in 1950.
CR Type: Inference
OA: IMO: E
Explanation: We're told that, from 1950 to 1980, the percentage of all skirelated injuries at ski resorts that did not occur on the slopes increased from
10% to 25%. That must mean that the percentage of ski-related injuries at
ski resorts that did occur on the slopes must have decreased over that same
period. Think about it this way: there are only two possible alternatives for a
skit related injury: either it occurs on the slopes or it does not occur on the
slopes. So if the share of ski-related injuries represented by one type of injury
goes
up, the share represented by the other type MUST go down. That's choice
(E).

In two months, the legal minimum wage in the country of Kirlandia will
increase from five Kirlandic dollars(KD5.00) Per hour to KD5.50 per hour.
Opponents of this increase have argued that the resulting rise in wages will
drive the inflation rate up. In fact its impact on wages will probably be
negligible, since only a very small proportion of all Kirfandic workers are
currently receiving less than KD5.50 per hour.
Which of the following, if true, most seriously weakens the argument?

A. Most people in kirlandia who are currently earning the minimum wage
have been employed at their current jobs for less than a year.
B. Some firms in Kirlandia have paid workers considerably less than KD5.00
per hour, in violation of kirlandic employment regulations.
C. Many businesses hire trainees at or near the minimum wage but must
reward trained workers by keeping their paylevels above the pay level of
trainees.
D. The greatest growth in Kirlandia's economy in recent years has been in
those sectors where workers earn wages that tend to be much higher than
the minimum wage.
E The current minimum wage is insufficient for a worker holding only one job
to earn enough to support a family, even when working full time at that job.

Conclusion: "There will be no impact on wages although the minimum wage


in the country will increase from 5.00 to 5.50 per hour"
OA : C
Evidence: "only a very small proportion of all Kirlandic workers are currently
receiving less than KD5.50 per hour"
Assumption: The only impact of the new minimum wage will be at WAGES,
but no other issue (such as cost of living, for example)
We must look for something implying that there are MANY workers who are
going to be affected by the new minimum wage, that is workers that are
going to be paid more.
A) Out of scope. We do not care about the time; we care if the workers are
been paid the minimum wage or not
B) Out of scope. If the companies did not comply with the law before, they
will not comply with the new wage either
C) Can be
D) Out of scope. We do not care about the sectors
E) Out of scope. We do not care if the minimum wage is enough or not. We
only care if many people are going to be affected with the new minimum
wage.

When 100 people who have not used cocaine are tested for cocaine use, on
average only 5 will test positive. By contrast, of every 100 people who have
used cocaine 99 will test positive. Thus, when a randomly chosen group of
people is tested for cocaine use, the vast majority of those who test positive
will be people who have used cocaine.
A reasoning error in the argument is that the argument
(A) attempts to infer a value judgment from purely factual premises

(B) attributes to every member of the population the properties of the


average member of the population
(C) fails to take into account what proportion of the population have used
cocaine
(D) ignores the fact that some cocaine users do not test positive
(E) advocates testing people for cocaine use when there is no reason to
suspect that they have used cocaine.
OA: C IMO: B
I've seen a few similar questions about 'false positives' and 'false negatives'.
You can imagine the following situation:
100 people in Country X use cocaine
1,000,000 people in Country X do not use cocaine
Test all of these people, and 99 of the 100 cocaine users will test positive,
while 50,000 of the non-users will test positive. You have 50,099 people in
total who test positive, but of those, only 99/50,099 ~ 0.2 % are actually
cocaine users.
Without knowing something about cocaine use in the population as a whole,
we can't say very much about the people who will test positive. C is certainly
the answer

Famous personalities found guilty of many types of crimes in well-publicized


trials are increasingly sentenced to the performance of community service,
though unknown defendants convicted of similar crimes almost always serve
prison sentences. However, the principle of equality before the law rules out
using fame and publicity as relevant considerations in the sentencing of
convicted criminals.

The statements above, if true, most strongly support which one of the
following conclusions?
A. The principle of equality before the law is rigorously applied in only a few
types of criminal trials.
B. The number of convicted celebrities sentenced to community service
should equal the number of convicted unknown defendants sentenced to
community service.
C. The principle of equality before the law can properly be overridden by
other principles in some cases.
D. The sentencing of celebrities to community service instead of prison
constitutes a violation of the principle of equality before the law in many
cases.
E. The principle of equality before the law does not allow for leniency in
sentencing.

OA: IMO: E
http://www.beatthegmat.com/what-is-the-solution-cr-question-lawt63819.html

Corporate Officer: Last year was an unusually poor one for our chemical
division, which has traditionally contributed about 60 percent of the
corporation's profits. It is therefore encouraging that there is the following

evidence that the pharmaceutical division is growing stronger: it contributed


45 percent of the corporation's profits, up from 20 percent the previous year.
On the basis of the facts stated which of the following is the best critique of
the evidence presented above?
(A) The increase in the pharmaceutical division's contribution to corporation
profits could have resulted largely from the introduction of single, important
new product.
(B) In multidivisional corporations that have pharmaceutical divisions, over
half of the corporation's profits usually come from the pharmaceuticals.
(C) The percentage of the corporation's profits attributable to the
pharmaceutical division could have increased even if that division's
performance had not improved.
(D) The information cited does not make it possible to determine whether the
20 percent share of profits cited was itself an improvement over the year
before.
(E) The information cited does not make it possible to compare the
performance of the chemical and pharmaceutical divisions in of the percent
of total profits attributable to each.
OA: C; IMO: B
Explanation: I find the phrasing of the original question unusual - it asks for a
'critique of the evidence presented'. Evidence is factual; you can't offer a
critique of it. You can offer a critique of the interpretation of that evidence, or
of a conclusion drawn from that evidence. I assume that the question is
really asking us to find a flaw in the conclusion here, rather than asking us to
find a 'critique of the evidence', something which doesn't make sense.
With that interpretation, C is certainly correct. We may have had the
following:
Previous year:
Pharmaceutical Division: $20m profit
Company Total: $100m profit
Pharmaceutical Division: 20% of total profit
Last year:
Pharmaceutical Division: $4.50 profit
Company Total: $10 profit
Pharmaceutical Division: 45% of total profit
From the above example, we can see that the Pharmaceutical Division may
have performed much worse last year than the previous year, while still
accounting for a greater percentage of the overall profits of the company.

The information provided is not necessarily 'encouraging' news about the


Pharmaceutical Division at all.
Advertisers are often criticized for their unscrupulous manipulation of
peoples tastes and wants. There is evidence, however, that some
advertisers are motivated by moral as well as financial considerations. A
particular publication decided to change its image from being a family
newspaper to concentration on sex and violence, thus appealing to a
different readership. Some advertisers withdrew their advertisements from
the publication, and this must have been because they morally disapproved
of publishing salacious material.
Which one of the following, if true, would most strengthen the argument?
(A) The advertisers switched their advertisements to other family
newspapers.
(B) Some advertisers switched from family newspapers to advertise in the
changed publication.
(C) The advertisers expected their product sales to increase if they stayed
with the changed publication, but to decrease if they withdrew.
(D) People who generally read family newspapers are not likely to buy
newspapers that concentrate on sex and violence.
(E) It was expected that the changed publication would appeal principally.

CR Type: Strengthen
OA: C; IMO: E/C
Time : 5:00
My Take: Some adv. withdrew their adds. From publication = morally
disapproved of publishing salacious material.
Conclusion: Some adv. Motivated = moral + financial considerations.
(To show in some way that Adv. Were benefitted financially from changing
publication).
(A)The advertisers switched their advertisements to other family
newspapers.
Nowhere is it mention about the financial benefits.
(B) Some advertisers switched from family newspapers to advertise in the
changed publication.
Same as A
(C) The advertisers expected their product sales to increase if they stayed
with the changed publication, but to decrease if they withdrew.

Financial benefit is mention here.


(D) People who generally read family newspapers are not likely to buy
newspapers that concentrate on sex and violence.
People switch is not important.
(E) It was expected that the changed publication would appeal principally to
those in a different income group.
Different income group doesnt mean that Adv. Will be having more
money or less money
The Japanese economic model created strong domestic industries through
subsidies from its Ministry of Trade and by closing off competitive foreign
firms to its domestic market. This strategy promised to help economic growth
by incubating domestic industries. New Japanese industries could count on a
known local demand and would be protected from competition by tariffs and
other barriers. The program could reduce the amount of imports and
therefore improve the nation's balance of trade.
Which of the following, based on the passage above, is a weakness in this
economic strategy?
A. A protectionist policy will create animosity among other nations.
B. Fast growth of small industries will create a class of millionaires and
increase the inequality of income.
C. Subsidies and import constraints keep domestic prices high and impose a
burden on consumers.
D. Quotas are more regressive than tariffs.
E. The demand for the products made by the incubated industries would not
be known.

CR Type: Weaken
OA: C; IMO: E
Time: 3:11
Take from students: but C is better than E bcz stmt says that domestic
industry count on local demand(means demand will be supportive) and E
simply says that demand would not be known...it might be high(In this case
strengthen) or it might be low(in this case weaken the stmt)...so
inconclusive.

C. These domestic industries could in fact be hurt because consumers will

not be willing to buy the goods at such a high price. Or, if the consumers do
buy the goods, they will not buy as many products as they bought before.

According to a recent magazine article, of those office employees who


typically work 8 hours at the office each day but sometimes say that they will
work at home on a particular day, 25 percent actually work less than one
hour. At the same time, over 90 percent of those same office employees
believe they are more productive working at home than working in their
office.
The statements above, if true, best support which of the following
conclusions about the office employees discussed in the article?
A. On average, the office employees working at home for a day work fewer
hours than office employees working at the office.
B. 10 percent of the office employees are less productive working from home
than working in their office.
C. At least 15 percent of the office employees do not define productivity
exclusively in terms of the number of hours worked.
D. At least 25 percent of the office employees can complete the same
amount of work in one hour at home as in 8 hours at the office.
E. Some of the office employees make statements regarding their
productivity that are not in fact true
CR Type: Inference
Time: 2:00
OA:C
Find the reason properly

Most disposable plastic containers are now labeled with a code number (from
1 to 9) indicating the type or quality of the plastic. Plastics with the lowest
code numbers are the easiest for recycling plants to recycle and are thus the
most likely to be recycled after use rather than dumped in landfills. Plastics
labeled with the highest numbers are only rarely recycled. Consumers can
make a significant long-term reduction in the amount of waste that goes
unrecycled, therefore, by refusing to purchase those products packaged in
plastic containers labeled with the highest code numbers.
Which one of the following, if true, most seriously undermines the conclusion
above?
(A) The cost of collecting, sorting, and recycling discarded plastics is
currently higher than the cost of manufacturing new plastics from virgin
materials.
(B) Many consumers are unaware of the codes that are stamped on the
plastic containers.
(C) A plastic container almost always has a higher code number after it is
recycled than it had before recycling because the recycling process causes a
degradation of the quality of the plastic.
(D) Products packaged in plastics with the lowest code numbers are often
more expensive than those packaged in the higher-numbered plastics.
(E) Communities that collect all discarded plastic containers for potential
recycling later dump in landfills plastics with higher-numbered codes only
when it is clear that no recycler will take them.
CR Type: Weaken
OA: IMO: C
Time: 4:38
My Take: Conclusion:
Refusing to purch. Plastic Prod. Of high code no.

Cons = sig long-term reduction of waste that goes


unrecycled
(A) Eliminate: Out of scope. Who cares about the cost, we're talking about
harming the environment.
(B) Eliminate: Still consistent with info given and thus irrelevant. If "most"
people are aware, they could know which plastic product is of high code
number or low quality.
(C) Should people keep purchasing the lowered numbered plastics, there
would be a net increase in higher numbered plastics as more lowered
numbered plastics are bought and recycled. Therefore, the environment is
still harmed. C is correct.
(D) Eliminate: Out of scope. Again, costs.
(E) Eliminate: Still consistent with info given, thus irrelevant. If consumers
are not purchasing the highernumbered plastics, there won't be much to
dump anyways
The report released by the interior ministry states that within the past 5
years the national land-reclamation program has created a 19 percent
increase in arable land within the country. If these figures are accurate, the
program has been a huge success. Senator Cox, a distinguished
mathematician and a woman of brilliance, maintains, however, that the
reclamation program could not possibly have been successful. Clearly,
therefore, the figures cited in the report cannot be accurate.
The argument above exhibits an erroneous pattern of reasoning most similar
to that exhibited by which one of the following?
A. Albert's father claims that Albert does not know where the spare car keys
are hidden. Yesterday however, Albert reported that he had discovered the
spare car keys in the garage toolbox, so his father's claim cannot be true.
B. Gloria's drama teacher claims that her policy is to give each student the
opportunity to act in at least one play during the year but, since Gloria, who
attended every class, reports that she was not given such an opportunity the
teacher's claim cannot be true.
C. Amos claims that he can hold his breath under water for a full hour. Dr.
Treviso, a cardiopulmonary specialist, has stated that humans are
physiologically incapable of holding their breath for even half that long; so
Amos' claim cannot be true.
D. Evelyn reports that she got home before midnight. Robert, who always
knows the time, insists that she did not. If Robert is right, Evelyn could not

possibly have listened to the late news; since she admits not having listened
to the late news, her report cannot be true.
E. Moira, after observing the finish of the 60-kilometer bicycle race, reports
that Lee won with Adams a distant third. Lomas, a bicycle engineering
expert, insists, however, that Lee could not have won a race in which Adams
competed; so Moira's report cannot be true.
Find the reason or do it again you got if WRONG

Calorie restriction, a diet high in nutrients but low in calories, is known to


prolong the life of rats and mice by preventing heart disease, cancer,
diabetes, and other diseases. A six-month study of 48 moderately
overweight people, who each reduced their calorie intake by at least 25
percent, demonstrated decreases in insulin levels and body temperature,
with the greatest decrease observed in individuals with the greatest
percentage change in their calorie intake. Low insulin level and body
temperature are both considered signs of longevity, partly because an earlier
study by other researchers found both traits in long-lived people.
If the above statements are true, they support which of the following
inferences?
A. Calorie restriction produces similar results in humans as it does in rats and
mice.
B. Humans who reduce their calorie intake by at least 25 percent on a longterm basis will live longer than they would have had they not done so.
C. Calorie intake is directly correlated to insulin level in moderately
overweight individuals.

D. Individuals with low insulin levels are healthier than individuals with high
insulin levels.
E. Some individuals in the study reduced their calorie intake by more than 25
percent.
Answer: E
Explanation: In an inference question, we need to find a choice that must be
true based on one or more facts in the passage. The second sentence states:
"A six-month study of 48 moderately overweight people, who each reduced
their calorie intake by at least 25 percent, demonstrated decreases in insulin
levels and body temperature, with the greatest decrease observed in
individuals with the greatest percentage change in their calorie intake."
If you connect the two bold portions, you will see that it must be true that
some experienced decreases of greater than 25 percent (otherwise, it
wouldn't make sense to speak of the "greatest decrease".)
Notice that valid inferences are not necessarily mind-boggling and don't
necessarily have to integrate the entire stimulus.

All of the best comedians have had unhappy childhoods. Yet, many people
who have had happy childhoods are good comedians, and some good
comedians who have had miserably unhappy childhoods are happy adults.
If the statements in the passage are true, which one of the following CANNOT
be true?
(A) The proportion of good comedians who had unhappy childhoods is
greater than the proportion of the best comedians who did.
(B) Some good comedians have had unhappy childhoods and are unhappy
adults.
(C) Most of the best comedians are happy adults.
(D) More good comedians have had unhappy childhoods than have had
happy childhoods.

(E) The proportion of comedians who are happy adults is higher than the
proportions who are unhappy adults.
CR Type: Inference
OA: A; IMO: B
Explanation: The answer must be A. The first sentence says "All of the best
comedians have had unhappy childhoods." Some of the good comedians
have had unhappy childhoods. Since all of the best comedians have had
unhappy childhoods, it must be impossible that there is a higher fraction of
good comedians with unhappy childhoods.
Choice C while tempting could be true: just because all of the best
comedians have had unhappy childhoods does not mean that any of them
are unhappy as adults.
No of Best Comedians: 10
No of Good Comedians with happiness in CH: 70
No of good comedians with unhappy CH: 20.
So now the proportion of Best comedians is less than proportion of
comedians with unhappy CH.
This contradicts option A.
-----------------------------------------------------as per the stimuli -- all the best comedians have unhappy childhood...
therefore, acc to the nos above best comedians with unhappy childhood =
10
so 10 in 10 best comedians have had unhappy childhood... this gives a
proportion of 100%
now you said there are 90 good comedians...70 with happy childhood and 20
with unhappy childhood
we are concerned only with 20 good comedians who have had unhappy
childhood
therefore proportion of good comedians who had unhappy childhood = 20/
(70+20)
= 20/90
Let us take another case .. say all the 90 good comedians had unhappy
childhoodnow the proportion of good comedians having unhappy childhood
= 90/90 = 100%

It can never be greater than the proportion of best comedians having


unhappy childhood
hence A...Key is Best comedians with UNHAPPY CHILDHOOD AND good
comedians WITH UNHAPPY CHILDHOOD...

Which of the following, if true, provides evidence that most logically


completes the argument below?
According to a widely held economic hypothesis, imposing strict
environmental regulations reduces economic growth. This hypothesis is
undermined by the fact that the states with the strictest environmental
regulations also have the highest economic growth. This fact does not show
that environmental regulations promote growth, however, since ______.

A. those states with the strictest environmental regulations invest the most
in education and job training
B. Even those states that have only moderately strict environmental
regulations have higher growth than those with the least-strict regulations
C. many states that are experiencing reduced economic growth are
considering weakening their environmental regulations
D. after introducing stricter environmental regulations, many states
experienced increased economic growth
E. even those states with very weak environmental regulations have
experienced at least some growth
CR Type: Inference
OA: A; IMO: E
Argument:
Hypothesis: strict environmental regulations results in reduced economic
growth
problem: states with strict regulations also have highest growth (no cause
effect given - just note that both are present. This is called a correlation they're present together but not necessarily connected by a cause-effect
relationship.)
The fact that they're correlated doesn't mean that environmental regs cause
economic growth.
So we need some reason to show that just because they're correlated
doesn't mean one causes the other. One way to do this is to have some other
thing that is likely to result in economic growth.
A) education and job training are likely to result in economic growth. Check.
B) this is just repeating the correlation we were already given in the
argument. No new info.
C) this is out of scope - it doesn't address the idea that something else might
be causing the growth instead of environmental regs
D) this would tend to reinforce the idea that environmental regs do result in
economic growth. That's the opposite of what we want to show.
E) this is also out of scope - it doesn't address the idea that something else
might be causing the growth instead of environmental regs

One state adds a 7 percent sales tax to the price of most products purchased
within its jurisdiction. This tax, therefore, if viewed as tax on income, has the

reverse effect of the federal income tax: the lower the income, the higher the
annual percentage rate at which the income is taxed.
The conclusion above would be properly drawn if which of the following were
assumed as a premise?
(A) The amount of money citizens spend on products subject to the state tax
tends to be equal across income levels.
(B) The federal income tax favors citizens with high incomes, whereas the
state sales tax favors citizens with low incomes.
(C) Citizens with low annual incomes can afford to pay a relatively higher
percentage of their incomes in state sales tax, since their federal income tax
is relatively low.
(D) The lower a state's sales tax, the more it will tend to redistribute income
from the more affluent citizens to the rest of society.
(E) Citizens who fail to earn federally taxable income are also exempt from
the state sales tax.
CR Type: Assumption
OA: A; IMO: C
Explanation: Now what is the conclusion?
This tax, therefore, if viewed as tax on income, has the reverse effect of the
federal income tax: the lower the income, the higher the annual percentage
rate at which the income is taxed.
Now if the amount that you pay is the same then what happens to the
percentage rate as it was discussed.
TAX/(low income) and TAX/(high income) the first one is definitely greater
than the second and it is the way in which I reasoned it out.
If we assume that people with different income levels spend different
amounts then the relation as mentioned in the stimulus (the lower the
income, the higher the annual percentage rate at which the income is taxed)
could not have been achieved.
So even if we take the different approach for an assumption question
(negating the choices to get a weakening option), A seems to be the best
one.
Another GUY : X has a salary of 10,000$ and Y has a salary of 5000$
They both decide to buy a cell phone worth 500$ let us assume sales tax for
the mobile is 100$
Now calculate the percentage impact on their salaries
X pays 1 % of his income as Stax
Y pays 2% of his income as Stax
Thus as a result with Y having less income gets to pay more % of his income
as tax.

A :says the amount that X spends totally on buyin things are same as that of
Y.(Meaning to say rich ppl will not buy more and dont end up payin more tax)

Which of the following most logically completes the passage?


Each species of moth has an optimal body temperature for effective flight,
and when air temperatures fall much below that temperature, the moths
typically have to remain inactive on vegetation for extended periods, leaving
them highly vulnerable to predators. In general, larger moths can fly faster
than smaller ones and hence have a better chance of evading flying
predators, but they also have higher optimal body temperatures, which
explains why ______.
A. large moths are generally able to maneuver better in flight than smaller
moths
B. large moths are proportionally much more common in warm climates than
in cool climates
C. small moths are more likely than large moths to be effectively
camouflaged while on vegetation
D. large moths typically have wings that are larger in proportion to their body
size than smaller moths do
E. most predators of moths prey not only on several different species of moth
but also on various species of other insects
CR Type: Inference
Time: 4:07
OA: A; IMO: B
My take: Each sp. Of moth =opt. body temp -> effective flight
OA: A
B. It is wrong because no where it is mention the temp. level of warm climate
Suppose small moth opt. temp = -5; large moth opt. temp = 0, then cold
climate temp = 0 then the condition satisfies ,but if cold is 10 then what ?

Mullen has proposed to raise taxes on the rich, who made so much money
during the past decade. Yet Mullens tax records show heavy investment in
business during that time and large profits; so Mullens proposal does not
deserve our consideration.
The flawed reasoning in the argument above is most similar to the flawed
reasoning in which one of the following?
(A) Do not vote for Smiths proposed legislation to subsidize child care for
working parents; Smith is a working parent.
(B) Do not put any credence in Dr. Hans recent proposal to ban smoking in
all public places; Dr. Han is a heavy smoker.
(C) The previous witnesss testimony ought to be ignored; he has been
convicted of both forgery and mail fraud.
(D) Board member Timms proposal to raise the salaries of the companys
middle managers does not deserve to be considered; Timms daughter is a
middle manager at the companys headquarters.
(E) Dr. Wasows analysis of the design of this bridge should not be taken
seriously; after all, Dr. Wasow has previously only designed factory buildings.

C. [ loss of credibility is not case in the passage, it is mere involvement ]


D. [ this says: not consider because someone related to Timm is involved.
Timm shud be involved in what he is proposing]
E. [ incompetency is not the case with Mullen above]
now that leaves A and B, I chose B because that has the negative
connotation. Dan is a heavy smoker so doesnt put credance into what he
says. [ Because of person rather than intellect about what he says]
A doesn't have a negative aspect. the person is a parent and is proposing
subsidy for child care. no attack on the person (unless u consider parenting
to be bad)

There is relatively little room for growth in the overall carpet market, which is
tied to the size of the population. Most who purchase carpet do so only once
or twice, first in their twenties or thirties, and then perhaps again in their
fifties or sixties. Thus as the population ages, companies producing carpet
will be able to gain market share in the carpet market only through
purchasing competitors, and not through more aggressive marketing.
Which one of the following, if true, casts the most doubt on the conclusion
above?
(A) Most of the major carpet producers market other floor coverings as well.
(B) Most established carpet producers market several different brand names
and varieties, and there is no remaining niche in the market for new brands
to fill.
(C) Two of the three mergers in the industrys last ten years led to a decline
in profits and revenues for the newly merged companies.
(D) Price reductions, achieved by cost-cutting in production, by some of the
dominant firms in the carpet market are causing other producers to leave the
market altogether.
(E) The carpet market is unlike most markets in that consumers are
becoming increasingly resistant to new patterns and styles

CR Type: Weaken Type


OA: IMO: D
Time: 4:01 worst
My Take: Concl: As population ages -> Inc. in market share of carpet market
only through purchasing competitors and not through more aggressive
marketing.
So try to find that aggressive marketing caused the increase in market share

Czannes art inspired the next generation of artists, twentieth-century


modernist creators of abstract art. While most experts rank Czanne as an
early modernist, a small few reject this idea. Franoise Cachin, for example,
bluntly states that such an ascription is overplayed, and says that
Czannes work is too often observed from a modern point of view.
Which one of the following statements is most strongly supported by the
information above?
(A) Czannes work is highly controversial.
(B) Czanne was an early creator of abstract art.
(C) Czannes work helped to develop modernism.
(D) Modern art owes less to Czanne than many experts believe.
(E) Czannes work tends to be misinterpreted as modernist.

CR Type: Inference
OA: C; IMO: E
(A) Czannes work is highly controversial.

Most experts rank him as a modernist but a small few reject this idea. So
the idea that he is "highly controersial" could easily be false.
(B) Czanne was an early creator of abstract art.
We are told that the experts themselves are in disagreement about this
idea. Therefore, it could easily be false.
(C) Czannes work helped to develop modernism.
Because Cezanne's art "inspired...the modernist creators of abstract art"
this choice must be true.
At this point, we would stop and choose C, but let's look at the other
choices. Choices D and E can be eliminated in one fell swoop: although we
learn what most experts believe and that a small few disagree with them,
we never actually learn the author's position. Therefore, these choices
could easily be false.

How can 'C' be true? Isnt that objected by a few people?


the idea that he WAS a modernist (that he "ranked" as one) is what is
disputed by some; not the idea that he influenced (=inspired) modernism.

If there are more oak trees in Oregon than there are leaves on any one
Oregon oak tree, and if every Oregon oak tree has at least one leaf, then
__________.
Which of the following most logically completes the passage?
A. the average number of oak leaves per Oregon oak tree must be less than
half the number of Oregon oak trees
B. there are fewer leaves on at least one Oregon oak tree than half the
number of those trees
C. there must be at least two oak trees in Oregon with the same number of
leaves
D. there must be at least as many Oregon oak trees with half as many leaves
as the Oregon tree with the most leaves, as there are Oregon oak trees with
twice as many leaves as the Oregon oak tree with the fewest leaves
E. there must be more oak trees than any other type of tree in Oregon
CR Type: Inference
OA: C; IMO: A
Justification:
Strategy: Try to make the problem more solvable by simplifying. First, delete
the words oak and Oregon. This helps remove unnecessary clutter.
Second, try to solve by working with a finite number of trees and leaves:
If we have four trees, then each tree has at least one leaf but no more than
three.
(C) is true for any number of trees. For example, if there are three trees, one
tree may have one leaf, another tree may have two leaves, but if the third
tree had either three or no leaves, then that would contradict the initial
statement. The third tree must have either one or two leaves (i.e., at least
two trees must have the same number of leaves).
(A) is not necessarily true; if, for instance, we have four trees, each of them
could have three leaves.
(B) is impossible if there are only two trees because each tree must have at

least one leaf (as weird as it may sound to have just two trees in Oregon!)
(D) isnt necessarily true; for example, all trees could have the same number
of leaves. This is a typical example of confusing syntax.
(E) is out because there's no talk about any other tree type. We're strictly
interested in oak trees!

Wolves can double their numbers every year. They do not do so, however, if
the area in which they live is already full and the territory of each pack
borders on the territory of other packs on all sides. Lone wolves, unable to
establish a territory near their place of origin, disperse to a less desirable
habitat and often are hunted by people.
Which of the following conclusions may be properly drawn from the
information in the passage above?
A. The number of wolves in any area suitable for wolves may be predicted to
quadruple in two years.
B. Wolves are ejected from the pack to wander singly as lone wolves when
the number of wolves in the pack has doubled.
C. The amount of territory suitable for wolves has been diminishing in recent
years as a result of human encroachment, with a resulting diminution in the
wolf population.
D. Wolf hunting can continue at a moderate rate on the fringes of a good
habitat for wolves without a decrease in the average yearly wolf population
in that territory.
E. The wolf population density in a given area of wolf territory does not vary
with the yearly rate of wolf reproduction.
CR Type: Inference
OA: D IMO: E
Students: So let's see how we break down this argument. We know that the
wolf population of a certain area can double every year, but they won't
necessarily do that if the territory is too crowded. Lone wolves who can't get
their own territory will move to places that are not so good, where they may
be hunted by people.
A is not correct because it's not necessarily true, as stated in our first
premise: while they can double every year (they'd quadruple in two years),
this may not happen because of territory issues.
B can be eliminated because we're explicitly told in the argument that lone

wolves are "unable to establish a territory near their place of origin". This
does not mean that they're expelled from the pack just because the pack has
doubled - maybe some wolves are chased away from the pack before the
number doubles...
While we may feel like C is right because we know it's true from experience,
there's nothing in the argument to suggest human encroachment on the
wolves' territory.
D is actually the correct answer here. Even if some wolves are hunted down
by humans, since the pack can double every year, the loss of a few wolves
will be offset by the birth of new wolves. Take an example: suppose you start
out with a pack of 15 wolves in a certain territory. 7 of them became
aggressive and they had to be hunted down. We're left with 15 - 7 = 8
wolves in the pack. It's a great loss, but by the end of the year the pack can
double. So the new number of wolves will be 2*8 = 16 wolves, which is
pretty close to 15.
E kindda contradicts the passage. If the wolves double every year in a
certain territory that's not yet full, then the density gets bigger and bigger.

Throughout the 1950s, there were increases in the numbers of dead birds
found in agricultural areas after pesticide sprayings. Pesticide
manufacturers claimed that the publicity given to bird deaths stimulated
volunteers to look for dead birds, and that the increase in numbers
reported was attributable to the increase in the number of people looking.
Which of the following statements, if true, would help to refute the claim of
the pesticide manufacturers?
(A)The publicity given to bird deaths was largely regional and never
reached national proportions.
(B) Pesticide sprayings were timed to coincide with various phases of the
life cycles of the insects they destroyed.
(C)No provision was made to ensure that a dead bird would not be reported
by more than one observer.
(D) Initial increases in bird deaths had been noticed by agricultural workers
long before any publicity had been given to the matter.
(E) Dead birds of the same species as those found in agricultural areas had
been found along coastal areas where no farming took place.
CR Type: Weaken
OA: D; IMO: B
SEARCH FOR REASON .

Teenagers are often priced out of the labor market by the governmentmandated minimum wage level because employers cannot afford to pay
that much for extra help. Therefore, if Congress institutes a subminimum
wage, a new lower legal wage for teenagers, the teenage unemployment
rate, which has been rising since 1960, will no longer increase.
Which of the following, if true, would most weaken the argument above?
(A) Since 1960 the teenage unemployment rate has risen when the
minimum wage has risen.
(B) Since 1960 the teenage unemployment rate has risen even when the
minimum wage remained constant.
(C) Employers often hire extra help during holiday and warm weather
seasons.
(D) The teenage unemployment rate rose more quickly in the 1970s than it
did in the 1960s.
(E) The teenage unemployment rate has occasionally declined in the years
since 1960.
CR Type: Weaken
OA: B; IMO: C
SEARCH Students are not happy with the answer choice

Statement of a United States copper mining company: Import quotas should


be imposed on the less expensive copper mined outside the country to
maintain the price of copper in this country; otherwise, our companies will
not be able to stay in business.
Response of a United States copper wire manufacturer: United States wire
and cable manufacturers purchase about 70 percent of the copper mined in
the United States. If the copper prices we pay are not at the international
level, our sales will drop, and then the demand for United States copper will
go down.
If the factual information presented by both companies is accurate, the best
assessment of the logical relationship between the two arguments is that the
wire manufacturers argument
(A) is self-serving and irrelevant to the proposal of the mining company
(B) is circular, presupposing what it seeks to prove about the proposal of the
mining company
(C) shows that the proposal of the mining company would have a negative
effect on the mining companys own business
(D) fails to give a reason why the proposal of the mining company should not
be put into effect to alleviate the concern of the mining company for staying
in business
(E) establishes that even the mining companys business will prosper if the
mining companys proposal is rejected
CR type: Method of argument
OA: C; IMO: B
Take away: 70% domestic CU + 30% international CU = current cu cost is
lower

70% domestic CU + 30% international CU ( NOW more expensive) = overall


expense is more
which in turn might indicate that the demand is lowered, leading to an
overall reduction in consumption.

Continuous indoor fluorescent light benefits the health of hamsters with


inherited heart disease. A group of them exposed to continuous fluorescent
light survived twenty-five percent longer than a similar group exposed
instead to equal periods of indoor fluorescent light and of darkness.
The method of the research described above is most likely to be applicable in
addressing which of the following questions?
(A) Can industrial workers who need to see their work do so better by
sunlight or by fluorescent light?
(B) Can hospital lighting be improved to promote the recovery of patients?
(C) How do deep-sea fish survive in total darkness?
(D) What are the inherited illnesses to which hamsters are subject?
(E) Are there plants that require specific periods of darkness in order to
bloom?
CR type: Method of argument.
OA: B; IMO:E
Take away:
Consider the essentials of the research described in the prompt:
1. Exposure to fluorescent light is compared to exposure to both fluorescent
light and darkness.

2. This is done to study hamsters' survival from inherited heart disease.


The main issue is studying how a specific type of light affects recovery from
sickness.
The main problem with C is that we're not interested in how an animal
survives in darkness; we're interested in recovery from illness. Notice that
this answer choice deals with basic, everyday survival, not the specific
situation of illness. It also deals with darkness, whereas we're interested in
the effects of light.
E is tempting, but again, we're not interested in darkness, and we're not
interested in survival requirements. We're interested in increasing survival in
relation to having a disease.
Only B addresses the issue of how specific lighting might increase recovery
from sickness.
_________________

Lists of hospitals have been compiled showing which hospitals have patient
death
rates exceeding the national average. The data have been adjusted to allow
for differences in
the ages of patients.
Each of the following, if true, provides a good logical ground for hospitals to
object to interpreting rank on these lists as one of the indices of the
quality of hospital care EXCEPT:
(A) Rank order might indicate insignificant differences, rather than large
differences, in numbers of patient deaths.
(B) Hospitals that keep patients longer are likely to have higher death rates
than those that discharge patients earlier but do not record deaths of
patients at home after discharge.
(C) Patients who are very old on admission to a hospital are less likely than
younger patients to survive the same types of illnesses or surgical
procedures.
(D) Some hospitals serve a larger proportion of low-income patients, who
tend to be more seriously ill when admitted to a hospital.

(E) For-profit hospitals sometimes do not provide intensive-care units and


other expensive services for very sick patients but refer or transfer such
patients to other hospitals
CR Type: Except
IMO: A; OA: C
Explanation:
step 1: Read the question stem: the question is asking to pick the choice that
does NOT provide a basis to object using SOME lists (we haven't read the
body of the question yet) to measure hospital care quality.
Another way (an easier way) is to eliminate 4 choices that DO provide the
basis to question the use of these lists.
Step 2: Read the question body.
premise 1: lists contain hospitals that have higher than average patient
death rates.
premise 2: data adjusts for age of patients.
Step 3: think of possible reasons why the lists should not be used. (you may
not be able to capture all possible reasons, but the process of thinking about
the answers puts you on a fast track to eliminating the incorrect answers)
- majority of the patients treated in hospitals do not die.
- data does not adjust for economic/social/quality of life factors that may
influence mortality.
- Data also does not account for what kind of diseases hospitals treat for.
Hospitals receiving mostly cancer patients or severe trauma victims may
have higher mortality rates.
Step 4: Eliminate, eliminate, eliminate.
A: Hospital that are closely ranked may have large differences in death rates.
This one is a tough one to eliminate. Come back to it later.
B: early discharge does not capture patients who die later. So B is a good
reason to object. Eliminate B.
C: Data adjusted for age, so not a valid reason to object. Possible answer.
Still keep going and come back to it.

D: economic and quality of life issues may affect survival rate. Good reason
to object. Eliminate D.
E: Types of patients (seriously ill versus moderately ill) received may be
different. Good reason to object. Eliminate E.
Step 5: Choose the correct choice. Looking at A and C (the only choices not
eliminated in step 4), A does point out a possible flaw in evaluating ho good
a hospital is. It is not a strong basis for objection, but C just can not be a
basis for objecting to the lists because the lists are corrected for age
variation (as mentioned in the body).
So the correct answer is C.

In Patton City, days are categorized as having heavy rainfall (more than two
inches), moderate rainfall (more than one inch, but no more than two
inches), light rainfall (at least a trace, but no more than one inch), or no
rainfall. In 1990, there were fewer days with light rainfall than in 1910 and
fewer with moderate rainfall, yet total rainfall for the year was 20 percent
higher in 1990 than in 1910.
If the statements above are true, then it is also possible that in Patton City
A. the number of days with heavy rainfall was lower in 1990 than in 1910
B. the number of days with some rainfall, but no more than two inches, was
the same in 1990 as in 1910

C. the number of days with some rainfall, but no more than two inches, was
higher in 1990 than in 1910
D. the total number of inches of rain that fell on days with moderate rainfall
in 1990 was more than twice what it had been in 1910
E. the average amount of rainfall per month was lower in 1990 than in 1910
CR Type: Inference
OA: A; IMO: C
Stuarts Comment:
Before we even think about approaching the answer choices, we should
analyze the stimulus, determining everything that must be true (and what
does not necessarily have to be true). (Step 2 of Kaplan method for inference
questions).
There are four kinds of days:
1) days with no rainfall;
2) days with light rainfall, which is more than zero up to (and inlcuding) one
inch;
3) days with moderate rainfall, which is more than one and up to (and
including) two inches; and
4) days with heavy rainfall, which is anything more than two inches.
In 1990, there were fewer days with light and moderate rainfall than in 1910.
Also, in 1990, there was more total rainfall. This does not necessarily mean
that there were more heavy rainfall days in 1990. Remember, we're talking
ranges here. All of the light days in 1910 could have been close to zero while
all of the light days in 1990 could have been closer to one. So, the excess
rain in 1990 could be due either to more heavily rainy days or else the light
and moderate days being more "rainy" than they were in 1910.
...Therefore, choice A could be true or is possible (matches the above
deduction). Because we've found a match to our prediction, and because
there can be only one credited response, we are done. Seriously, you are
better off taking an extra moment confirming that this could be true, then
trying to prove that the other four choices must be false (or impossible). You
don't get rewarded for figuring four reasons why four wrong answers are
wrong. (Steps 3 and 4 are predict and match).
But let's look at the other choices:
B. the number of days with some rainfall, but no more than two inches, was
the same
in 1990 as in 1910.

These are the days with light and moderate rainfall. There were more of
these days in 1910 than in in 1990. Therefore, this must be false
(impossible).
C. the number of days with some rainfall, but no more than two inches, was
higher in
1990 than in 1910.
This choice must be false for the same reason that chocie B must be false.
D. the total number of inches of rain that fell on days with moderate rainfall
in 1990
was more than twice what it had been in 1910.
This is mathematically impossible. We know there are fewer moderatley
rainy days in 1990 than in 1910, and the range is less than one inch. Even if
we were to maximize moderate rainy days in 1990 and minimize in 1910,
and minimize rain per moderately rainy day in 1910 while maximizing rain
per moderately rainy day in 1990, this is still impossible. For example, let's
say there are 11 moderately rainy days in 1910 with 1.1 inch per day, and 10
moderately rainy days in 1990 with 2.0 inches per day. Then we would have
11*1.1 = 12.1 inches of rainfall on moderately rainy days in 1910 while we
would have 10*2.0 = 20 inches of rainfall on moderately rainy days in 1990.
It is impossible that the rainfall on moderately rainy days in 1990 is more
than twice as great as the amount of rain that fell on moderately rainy days
in 1910.
Therefore, choice D must be false, or is impossible.
(But see how long that took to figure out? Honestly, process of elimination is
not efficient).
Finally, choice E:
E. the average amount of rainfall per month was lower in 1990 than in 1910
Because the total amount of rain in 1990 was greater than it was in 1910,
then the average rainfall per month will also be greater. For example, if total
rainfall in 1910 was 100 and in 1990 was 120 (20% greater), then obviously
120/12 is greater than 100/12. Therefore, this choice also must be false
(impossible).
Choose A.

The fishing industry cannot currently be relied upon to help the government
count the seabirds killed by net fishing, since an accurate count might result

in restriction of net fishing. The government should therefore institute a


program under which tissue samples from the dead birds are examined to
determine the amount of toxins in the fish eaten by the birds. The industry
would then have a reason to turn in the bird carcasses, since the industry
needs to know whether the fish it catches are contaminated with toxins.
Which one of the following, if true, most strongly indicates that the
government program would not by itself provide an accurate count of the
seabirds killed by net fishing?
(A) The seabirds killed by net fishing might be contaminated with several
different toxins even if the birds eat only one kind of fish
(B) The fishing industry could learn whether the fish it catches are
contaminated with toxins if only a few of the seabirds killed by the nets were
examined
(C) The government could gain valuable information about the source of
toxins by examining tissue samples of the seabirds caught in the nets.
(D) The fish caught in a particular net might be contaminated with the same
toxins as those in the seabirds caught in that net.
(E) The government would be willing to certify that the fish caught by the
industry are not contaminated with toxins if tests done on the seabirds
showed no contamination
CR Type: Strengthen
OA: B; IMO D
Explanation: Here, we have a proposal--which, per the Kaplan method, we
paraphrase: "We will get an accurate count of seabirds by testing them for
toxins." On the GMAT, predictions and proposals assume that they are viable
on their own terms; we aren't looking for unrelated evidence that might or
might not affect the conclusion, but rather for something that explains why
testing birds for toxins will not lead to an accurate count. B says that testing
birds for toxicity can be accomplished with only a few seabirds; in other
words, there is no reason for testing to give an accurate count. It is our
answer

Emissions from automobiles that burn gasoline and automobiles that burn
diesel fuel are threatening the quality of life on our planet, contaminating
both urban air and global atmosphere. Therefore, the only effective way to
reduce such emissions is to replace the conventional diesel fuel and gasoline
used in automobiles with cleaner-burning fuels, such as methanol, that
create fewer emissions.
Which one of the following is an assumption on which the argument
depends?
(A) Reducing the use of automobiles would not be a more effective means to
reduce automobile emissions than the use of methanol.
(B) There is no fuel other than methanol that is cleaner-burning than both
diesel fuel and gasoline.
(C) If given a choice of automobile fuels, automobile owners would not select
gasoline over methanol.
(D) Automobile emissions constitute the most serious threat to the global
environment.
(E) At any given time there is a direct correlation between the level of urban
air pollution and the level of contamination present in the global atmosphere.
CR Type: Assumption
OA: A; IMO: B
Explanation
(A)
The argument states that the only effective way to reduce the amount of
automobile emissions which plague our atmosphere is to switch to cleanerburning fuels. But what if
people just reduced their amount of driving? Wouldnt that also reduce the
amount of car emissions in the atmosphere? Use the Denial Test on (A): If a
reduction in automobile use
were more effective than the switch in fuels, then the conclusion that
cleaner-burning fuels are the only effective way of reducing emissions
would fall apart.
(B) So what if there were a fuel besides methanol thats cleaner-burning than
diesel fuel and gasoline? This wouldt affect the argument. The author
doesnt specifically advocate a
switch to methanol; she calls for the use of any cleaner-burning alternative
fuels, such as methanol.
(C) The passage doesnt suggest offering drivers a choice of fuels, so their
preference is immaterial.
(D) Not necessary. The existence of other, more serious threats to the
environment wouldnt change the fact that automobile emissions are

threatening the quality of life on


our planet.
(E) is beyond the scope. The passage tells us that automobile emissions
contaminate both the urban air and the global atmosphere. How the two are
related doesnt affect the
argument.
To check your answer in an assumption question, try negating it. If you take
the assumption to be false, and the argument still works, look for another
choice.
Pick up on extreme words like only, always, never, and so on. An
argument that concludes a specific practice is the only way to accomplish
something usually
depends on the assumption that all other means are impossible or
ineffective.

In the past 50 years, the population of honey bees in the united states has
been cut in half. The decline is due primarily to the increasing use of
pesticides in the united states, as well as to the introduction of two types of
mites that weaken and kill the bees. Honeybees are the primary pollinators
of a variety of important fruit crops, including oranges, apples, grapes,
peaches, cranberries and watermelons. Therefore, if the honey bee
population continues this drastic decline, then most fruits will no longer be
available to consumers.
In evaluating the conclusion, which of the following questions would be least
useful to answer?
1. Are there other insects pollinators that could pollinate these fruit crops
instead of the honeybee?
2. Are honeybee populations declining in other important fruit- producing
regions, like Chile and New Zealand?
3. Is it feasible for humans to hand-pollinate the fruits that have been
pollinated by bees?
4. Will reducing the use of pesticides in the united states reverse the
decline in the honeybee population?
5. Is it possible to genetically engineer fruit-producing plants so that they
no longer require pollination?

Passenger boardings on trains almost doubled between 1985 and 1995,


yet the actual number of trips made by passengers increased by only 38
percent over the same period. The reason for this discrepancy is that at
least two boardings are counted when a passenger must take more than
one train trip to get to the final destination of his or her trip.
Which of the following, if true, best helps explain why the increase in
boardings exceeded the increase in the number of train passengers
between 1985 and 1995?
A. Between 1985 and 1995, the number of train stations increased
dramatically.
B. Between 1985 and 1995, the number of train lines decreased
significantly.
C. Between 1985 and 1995, the proportion of train trips requiring
passengers to change trains en route to their final
destinations increased significantly.
D. The proportion of business travelers using trains increased significantly
between 1985 and 1995 relative to the
proportion of pleasure travelers.
E. The average seating capacity of passenger trains increased
significantly between 1985 and 1995.
http://www.beatthegmat.com/cr-paradox-hard-to-digestt75114.html#337750

Thousands of people have tonsillectomies every year and all live normal lives after the operation.
We can conclude, from this observation, that the tonsils have no function in the body.
The argument would be most weakened by which of the following, if it were true?
A. People live normal lives after appendectomies but the appendix is known to be part of the
digestive system.
B. Another part of the body can take over the function of the tonsils if they are removed.
C. The tonsils have been shown to have a vital role to play in the physiology of laboratory rabbits
and guinea pigs.
D. The human tonsil develops as part of the immune system, a system of vital importance in
defense against disease.
E. Tonsillectomies are performed only when the tonsils become seriously infected.
Correct Answer: B
Explanation:
The conclusion states that the tonsils have no function in the body. To weaken this conclusion we
should try to show that the tonsils do have a function, at least at some point in human life.
Comparison with another organ is not really useful, as other organs might or might not have a
demonstrable function, and might be very different from tonsils, and what happens in other
animals is also not directly relevant. Thus we can discard A and C. The statement in E is clearly
irrelevant in deciding whether tonsils have a function. So that leaves B and D. An organ might
develop as part of a system and yet have no function (like the appendix or muscles that wiggle
the ear), and so D is not convincing, whereas B suggests that there is a function even though this
function can be taken over by another part of the body. Thus B is the answer.

Scientists investigating a rare metabolic disorder hypothesized that obesity was a predisposing
factor in the development of the disease. A study of twenty patients found that, on average, the
patients were close to the normal weight for their height.
Before concluding that obesity is not a predisposing factor, the researchers would find the answer
to which of the following questions most useful?
A. Are the patients above or below normal height?
B. Were any of the patients underweight when the disorder was diagnosed?
C. Does weight loss reduce the severity of the symptoms?
D. Have the patients always been close to the normal weight for their heights?
E. How many of the patients had obese parents?
Correct Answer: D
Explanation:
The hypothesis that obesity is a factor in the development of the disease cannot really be tested
by looking at the current weights of the patients. They might have lost weigh recently (for any
reason such as doctors advice or even illness). Thus before concluding that obesity is NOT a
factor in the development we need to know about their weights before the onset of the disease.
Answer D is closest to this suggestion. In answer choice A the height is irrelevant as is the
parental weight in E. In B, the weight at diagnosis is already too late to infer anything about the
development of the disease. And in C what happens after the disease has developed is also
irrelevant.

Of the people who moved from one state to another when they retired, the
proportion who retired to SunState has decreased by 10 percent over the past
five years. Since many local businesses in SunState cater to retirees, this decline
is likely to have a noticeably negative economic effect on these businesses.
Which of the following, if true, most seriously weakens the argument?
a. SunState attracts more people who move from one state to another when
they retire than does any other state.
b. There are far more local businesses in SunState that cater to tourists than
there are local businesses that cater to retirees.
c. The number of retirees who have moved out of SunState to accept reemployment in other states has increased over the past five years.
d. SunState has lower property taxes than any other state, making the state a
magnet for retirees.
e. The total number of people who retired and moved to another state for their
retirement has increased significantly over the past five years.
The answer is E. Try playing with numbers.
Say five years ago there were 1,000 retirees who changed states and 30% went
to SunState. That would mean 300 to SunState.
Assume this year there were 2,000 retirees who changed states and only 27%
went to SunState. That would mean 540 to SunState.
This is a classic problem. Anytime you see proportions and absolutes mixed in a
problem you should always check to see whether that affects the answer.

Although well-known links exist between soda consumption and obesity, a recent
study has found that banning these sweetened beverages has no effect on
childrens eating habits. Since many school boards are fiercely debating such
policies, the results of this study should provide the hard data needed to make
an informed decision.
Which of the following, if true, most seriously undermines the argument?
a) Several studies show a link between banning sweetened beverages in school
and the eating habits of the affected children.
b) Childhood obesity is most effectively addressed when parents prevent their
children from drinking sweetened beverages both in and out of school.
c) School-age children consume more sugar from fatty foods than from
sweetened beverages.
d) Most of the school boards are considering banning sweetened beverages as a
remedy for misbehavior after lunch than for obesity.
e) Children of obese parents are more likely than children of non-obese parents
to become obese regardless of their food and beverage intake.
(A) is definitely tempting because it contradicts the opinion of the author.
However, this does not count as "weakening [or undermining] the argument"
on the GMAT. To weaken an argument, we have to attack its logic -- its
assumptions -- the relevance of the evidence to the conclusion. (A) simply
attacks the evidence and says "No, that isn't true." This will never be the
right answer to a GMAT question.
To attack the logic, we have to look for how the Evidence and Conclusion
differ. The Evidence is that, based on new studies, banning sodas does NOT
affect childhood eating habits. We assume this is true; the question isn't
whether it's true or not, but whether this evidence, taken as true, proves the
Conclusion. The Conclusion is that now, school boards will have "the hard
data needed to make an informed decision" -- that is, that based on these

new studies alone, the school board will definitely NOT ban sodas, because
there is proof that banning them will not affect eating habits.
The correct answer needs to say "Yes, that Evidence is true, but it doesn't
mean that the school board won't ban sodas; they may still ban them for
some other reason." (D) gives this other reason. If the school board isn't
planning to ban sodas in order to make children thinner, but only in order to
punish th echildren or stop them from misbehaving after lunch, then this
study is irrelevant to their debate.
Notice that the right answer doesn't weaken the author's ideas about the link
or lack thereof between soda and obesity or any of that; on the GMAT, it
rarely does, and doesn't ever have to. All the correct answer needs to do is to
say:
"Even if the given Evidence is true, that doesn't mean your Conclusion will be
true also."
And (D) says this, because given (D), even if they're convinced that soda
doesn't lead to obesity, they might still ban it for other reasons. It attacks the
logic of the Argument, not necessarily the Evidence or Conclusion
themselves.
Takeaways:
1) Never contradict the Evidence!
2) Look for anything that makes the specific Conclusion -- here, that the
school board won't ban soda -- less likely to come true even in light of the
current evidence.
3) Always think about the Logic of the Argument, not the factual details.
4) Always think about alternate reasons/causes/motivations/possibilities.

Between 1977 and 1989, the percentage of income paid to federal taxes by the richest one
percent of Americans decreased, from 40 percent to 25 percent. By the end of that same period,
however, the richest one percent of Americans were paying a larger proportion of all federal tax
revenues, from 12.7 percent in 1977 to 16.2 percent in 1989.
Which of the following, if true, contributes most to an explanation of the discrepancy described
above?
A. Between 1977 and 1989, the Internal Revenue Service increased the percentage of its staff
members responsible for audits and tax collection.
B. Between 1977 and 1989, the before-tax income of the richest one percent of Americans
increased by over 75 percent when adjusted for inflation.
C. Between 1977 and 1989, many of the richest one percent of Americans shifted their
investments from taxable to untaxable assets.
D. Between 1977 and 1989, the tax rate paid by middle-income Americans was reduced, but
several tax loopholes were eliminated.
E. Between 1977 and 1989, the amount of federal taxes paid by the richest one percent of
Americans increased by $45 billion, while the amount paid by all Americans rose by $50 billion.
B
Were told that from 1977 to 1989 the percentage of their own income that the richest one
percent of Americans paid to federal taxes decreased. At the same time, the proportion (or
percentage) of all federal tax revenues that was paid by these same rich Americans increased.
Were asked to clear up this apparent discrepancy: a lighter tax burden on the wealthy resulted
in their carrying more of the overall tax load. If, as (B) has it, the richest one percent are making
much more money than they once did, then the actual amount of money they pay in federal
taxes can increase, even though the percentage of their own income that this amount represents
decreases. This increased amount of taxes paid could represent an increased proportion of the
total federal tax revenues. (A) suggests that the IRS has increased its tax collecting efficiency,
but this is irrelevant to the question of how one percentage can increase while the other
decreases. (C) could explain how taxes on the rich account for more of the total tax revenues
(because more of their investments are taxable), but it doesnt explain why these increased

taxes account for a smaller percentage of their incomes. (D) raises new questions. If we assume
(which we cant) that the elimination of loopholes hurt the rich more than the tax cut helped
them, we could see how they might pay more taxes despite the cut in the top tax rate. But that
just presents us with the old problem: If theyre paying more taxes, how can they be paying less
of their income to taxes? On the other hand, if we assume (which we cant) that the tax cut in
(D) means theyre paying less in taxes, how can they be bearing more of the tax burden? On a
question asking for an explanation, avoid any choice that leaves you wondering. (E) shows how
the richest one percent account for a higher percentage of overall tax revenues. Theyve paid an
additional 45 billion dollars while everyone else has only had to come up with an additional 5
billionthe 5 billion left from the 50 billion after subtracting the 45 billion that the very rich paid.
However, this leaves out the other part of the dilemmait doesnt show how they can be paying
less of their income

Eight years ago hunting was banned in Greenfield County on the grounds
that hunting endangers public safety. Now the deer population in the county
is six times what it was before the ban. Deer are invading residential areas,
damaging property and causing motor vehicle accidents that result in serious
injury to motorists. Since there were never any hunting-related injuries in the
county, clearly the ban was not only unnecessary but has created a danger
to public safety that would not otherwise exist.
Which one of the following, if true, provides the strongest additional support
for the conclusion above?
(A) In surrounding counties, where hunting is permitted, the size of the deer
population has not increased in the last eight years.
(B) Motor vehicle accidents involving deer often result in damage to the
vehicle, injury to the motorist, or both.
(C) When deer populations increase beyond optimal size, disease and
malnutrition become more widespread among the deer herds.
(D) In residential areas in the county, many residents provide food and salt
for deer.
(E) Deer can cause extensive damage to ornamental shrubs and trees by
chewing on twigs and saplings.
OA: A
IMO: B
CR Type: Strengthening
Take: Conclusion: Ban = unnecessary and danger for public safety.

A.
B. The data is mentioned in the argument so this cant be right,
C. This weakens the argument.
D. Out of Scope
E. Out of scope.
Note: Answer choice mentioned in the argument cant be correct answer
choice.

Company X conducted a taste test to determine whether its new soft drink
had a good chance of commercial success. A sample of consumers was
asked to compare the flavor of the new soft drink to that of an established
brand without knowing the true identity of either beverage. Overwhelmingly,
the consumers preferred the taste of the proposed soft drink to that of the
established brand. Clearly, Company X has a good chance of commercial
success with its new soft drink.
Which of the following, if true, would most seriously weaken the argument
above?
A. Some of the consumers in the taste test preferred the flavor of the
established brand.
B. The other soft drink used in the taste test is also manufactured by
Company X.
C. The new soft drink will cost more than three times as much as any other
soft drink on the market.
D. Company X has not yet designed a label for the new soft drink.
E. The name of the new soft drink is very close to that of the established
brand.
OA: C
IMO: E
CR Type: weaken.

The conclusion of the argument is "Company X has a good chance of commercial


success with its new soft drink." Why? Because most consumers in the taste test
preferred its flavor to that of an established brand. We are asked to weaken the
conclusion.
(A) The fact that "some" preferred the established brand does not weaken the
argument, since we do not know how large a percentage "some" represents.
(B) The fact that Company X owns both brands does not weaken the claim that the
new brand will be successful.
(C) CORRECT. In order to weaken this argument, all we need to do is show that
there may be some reason to doubt that the flavor will be enough for the drink to be
successful. This choice states that the new drink will be much more expensive than
any other on the market. This does not prove that the drink will not be successful,
but it does give a reason to suspect that it might not be.
(D) Whether there is label for the new drink is not revelant to the claims of the
argument.
(E) The fact that the brands have similar names does not suggest that the new
brand will not be successful.

Note: If the name is very close that doesnt mean Company will not have a
chance of commercial success. Name proximity is just illusion/trap
Commerical success is affected by price.

Wide dissemination of wireless networks in cities is a practical way to meet


the needs of city households, schools and businesses. Rural communities
have found that wireless networks are both more reliable and cheaper than
land-based networks.
Which of the following would most likely be cited by a supporter of the
argument?
A. Urban areas do not pose additional problems for the effective operation of
wireless networks.
B. Wireless networks work far better where population density is low.
C. Iceland, a very rural country, successfully uses wireless networks.
D. The expenses of wireless transmission in areas with large buildings are
much higher.
E. Poor neighborhoods have less access to cable internet than do educators
or businesses
The argument claims that wide dissemination of wireless access is now a
practical way to meet urban needs, based on the evidence of its successful

use in rural areas. The author then must assume that urban areas provide no
additional problems for wireless use.
(A) CORRECT. This choice confirms an assumption of the argument and thus
strengthens the conclusion.
(B) This choice weakens the argument because it damages the assumption
that urban areas pose no extra problems for wireless use.
(C) This choice is irrelevant because it provides information about another
rural area; however, the conclusion concerns urban areas.
(D) This choice weakens the argument because it damages the assumption
that urban areas pose no extra problems for wireless use.
(E) This choice is an irrelevant distinction. The argument mentioned all three
groups as in need of this service. The suggestion that one group needs it
more than the others is irrelevant to the conclusion.

From 1980 to 1989, total consumption of fish in the country of Jurania


increased by 4.5 percent, and total consumption of poultry products there
increased by 9.0 percent. During the same period, the population of Jurania
increased by 6 percent, in part due to immigration to Jurania from other
countries in the region.
If the statements above are true, which of the following must also be true on
the basis of them?
A. During the 1980s in Jurania, profits of wholesale distributors of poultry
products increased at a greater rate than did profits of wholesale distributors
of fish.
B. For people who immigrated to Jurania during the 1980s , fish was less
likely to be a major part of their diet than was poultry.
C. In 1989 Juranians consumed twice as much poultry as fish.
D. For a significant of juranias population, both fish and poultry products
were a regular part of their diet during the 1980s.
E. Per capita consumption of fish in Jurania was lower in 1989 than in 1980.

OA: E.
From some student:
a) Statement on Profits is not relevant to the information given in the
stimulus
b) Not mentioned in stimulus
c) Absolute value of consumption of fish or poultry products is not mentioned
in stimulus
d) There could be other sources of food that forms the regular part of their
diet.
e) This is correct answer.
Population consumption of fish Per capita
1980 1000 100 units 0.1
1989 1060 104.5 units 0.09
Hence per capita consumption of fish is lower in 1989 than in 1980.

In 1992 outlaw fishing boats began illegally harvesting lobsters from the
territorial waters of the country of Belukia. Soon after, the annual tonnage of
lobster legally harvested in Belukian waters began declining; in 1996,
despite there being no reduction in the level of legal lobster fishing activity,
the local catch was 9,000 tons below pre-1992 levels. It is therefore highly
likely that the outlaw fishing boats harvested about 9,000 tons of lobster
illegally that year.
Which of the following is an assumption on which the argument depends?
A. The illegal lobster harvesting was not so extensive that the population of
catchable lobsters in Belukia's territorial waters had sharply declined by
1996.

B. The average annual lobster catch, in tons, of an outlaw fishing boat has
increased steadily since 1992.
C. Outlaw fishing boats do not, as a group, harvest more lobsters than do
licensed lobster-fishing boats.
D. The annual legal lobster harvest in Belukia in 1996 was not significantly
less than 9,000 tons.
E. A significant proportion of Belukia's operators of licensed lobster-fishing
boats went out of business between 1992 and 1996.
OA: A
Students explaination: The argument goes like so:
- illegal fishermen started operating in 1992 (premise)
- in 1996, the tonnage for legal fishermen was 9000 tons below pre-1992 levels, despite the fact
that legal fishing activity did not go down (premise)
- the only explanation is that poachers were fishing the 9000 tons (conclusion)
Assumptions are logical connections between the premises and the conclusion. It's useful to
remember that, if you negate an assumption, then the argument falls apart. The best answer
choice here is A, because negating A means that you've destroyed the argument: if indeed the
poaching was really big, then the lobster population perished, so the lower catch in 1996 can be
explained by the depletion of this resource and NOT by illegal fishing.
B may be true, but it is not an assumption. Think of the case when initial poaching was 1-2 tons
and now it is 10 tons. This does not add up to the 9000 tons in the argument.
C doesn't really matter, because we're not comparing them. We're only interested in the impact
over the legal fishermen and not how they stack up against poachers.
D doesn't matter either, because this info would only help us evaluate the current catch with the
loss.
E - is irrelevant as well, because we are told that no increase in activity occurred. If some went
out of business, then others must have taken their place for the activity to stay the same.

The total market value of real estate in Altonville has steadily declined over the past four
years. This decline has meant that the overall figure on which the citys property tax is
basedthe assessed value of that real estatehas also declined. Moreover, the
percentage of assessed value that was paid as property taxes to the city did not change
during this period.
The information above most strongly supports which of the following?
A. Money collected from property taxes provided most of Altonvilles revenue
during the past four years.
B. The percentage of Altonvilles overall revenue that was collected from property
taxes did not change over the past four years.
C. Altonville officials had proposed raising property tax rates during the past four

years but never did so because of strong opposition from property owners.
D. The total amount of revenue that Altonville has collected from property taxes has
declined over the past four years.
E. During the past four years, Altonville officials also did not increase tax rates on
other sources of revenue such as retail sales or business profits.
OA: C IMO : B
Check how and why

People in isolated rain-forest communities tend to live on a largely


vegetarian diet, and
they eat little salt. Few of them suffer from high blood pressure, and their
blood pressure
does not tend to increase with age, as is common in industrialized countries.
Such people
often do develop high blood pressure when they move to cities and adopt
high-salt diets.
Though suggestive, these facts do not establish salt as the culprit in high
blood pressure,
however, because ________.

A. genetic factors could account for the lack of increase of blood pressure
with age
among such people
B. people eating high-salt diets and living from birth in cities in industrialized
societies generally have a tendency to have high blood pressure
C. it is possible to have a low-salt diet while living in a city in an
industrialized
country
D. there are changes in other aspects of diet when such people move to the
city
E. salt is a necessity for human life, and death can occur when the body
loses too
much salt
understand the pattern though .,however
start with a fact : ppl in RFC live on Veggy diet + eat little salt + do not suffer from
HBP + BP doesnt increase with Age.
Contradiction : they develop HBP when move to cities + adopt high salt. But eating
high salt is not culprit because ?
We have to say something that goes against the fact given?
or say something that prove high salt doesnt cause HBP?
say something that its something else in cities that cause HBP?
Options:
A) --> Out of SCOPE.
B) --> liasing with Fact so Wrong.
C) --> irrelevant
D) --> some more ascpect of diet changes --> this may cause the HBP
E) --> so silly.

The traditional treatment of strep infections has been a seven-day course of


antibiotics, either penicillin or erythromycin. However, since many patients
stop taking those drugs within three days, reinfection is common in cases

where those drugs are prescribed. A new antibiotic requires only a three-day
course of treatment. Therefore, reinfection will probably be less common in
cases where the new antibiotic is prescribed than in cases where either
penicillin or erythromycin is prescribed.
Which of the following, if true, most strengthens the argument?
A. Some of the people who are allergic to penicillin are likely to be allergic to
the new antibiotic.
B. A course of treatment with the new antibiotic costs about the same as a
course of treatment with either penicillin or erythromycin.
C. The new antibiotic has been shown to be effective in eradicating bacterial
infections other than strep.
D. Some physicians have already begun to prescribe the new antibiotic
instead of penicillin or erythromycin for the treatment of some strep
infections.
E. Regardless of whether they take a traditional antibiotic or the new one,
most patients feel fully recovered after taking the drug for three days
OA: E
The text states that reinfection is common when patients use the former
treatments because they interrupt the 7-day treatment early (i.e. after 3
days). E makes it clear that with the new treatment, patients will likely take
the new drug for the entire 3 days that the drug needs in order to clear the
infection. This fact removes the following objection:
What if patients stop taking the new drug prematurely because they feel
better before the infection is fully cleared? Reinfection may still be common

In response to mounting pubic concern, an airplane manufacturer


implemented a program with the well-publicized goal of reducing by half the
total yearly amount of hazardous waste generated by its passenger-jet

division. When the program began in 1994, the divisions hazardous waste
output was 90 pounds per production worker; last year it was 40 pounds per
production worker. Clearly, therefore, charges that the manufacturers
program has not met its goal are false.
Which of the following is an assumption on which the argument depends?
A. The amount of nonhazardous waste generated each year by the
passenger-jet division has not increased significantly since 1994.
B. At least as many passenger jets were produced by the division last year as
had been produced in 1994.
C. Since 1994, other divisions in the company have achieved reductions in
hazardous waste output that are at least equal to that achieved in the
passenger-jet division.
D. The average number of weekly hours per production worker in the
passenger-jet division was not significantly greater last year than it was in
1994.
E. The number of production workers assigned to the passenger-jet division
was not significantly less in 1994 than it was last year.
OA: E
Take Away: 1994: Waste output = 90/production worker
Last year: Waste output= 40/production worker.
So no. of production worker is important.

The emission of sulfur dioxide when high-sulfur coal is burned is restricted by


law. New coal-burning plants usually comply with the law by installing

expensive equipment to filter sulfur dioxide from their emissions. These new
plants could save money by installing instead less expensive cleaning
equipment that chemically removes most sulfur from coal before
combustion.
Which of the following, if known, would be most relevant to evaluating the
claim above about how new coal-burning plants could save money?
A. Whether existing oil-burning plants are required to filter sulfur dioxide
from their emissions
B. Whether the expense of installing the cleaning equipment in a new plant
is less than the expense of installing the cleaning equipment in an older
plant
C. Whether the process of cleaning the coal is more expensive than the
process of filtering the emissions
D. Whether lawful emissions of sulfur dioxide from coal-burning plants are
damaging the environment
E. Whether existing plants that use the filtering equipment could replace this
equipment with the cleaning equipment and still compete with new plants
that install the cleaning equipment
OA:C
The emission of sulfur dioxide when high-sulfur coal is burned is restricted by law.
New coal-burning plants usually comply with the law by installing expensive
equipment to filter sulfur dioxide from their emissions. These new plants could save
money by installing instead less expensive cleaning equipment that chemically
removes most sulfur from coal before combustion.
Which of the following, if known, would be most relevant to evaluating the claim
above about how new coal-burning plants could save money?
a. Whether existing oil-burning plants are required to filter sulfur dioxide from their
emissions---does not say anything abt new plants.
b. Whether the expense of installing the cleaning equipment in a new plant is less
than the expense of installing the cleaning equipment in an older plan
comparision bet old and new not relevant
c. Whether the process of cleaning the coal is more expensive than the process of
filtering the emissions.
correct ans as this is known then it does support the claim that cleaning coal will
save money than installing filters

d. Whether lawful emissions of sulfur dioxide from coal-burning plants are damaging
the environment
does not tell abt saving money
e. Whether existing plants that use the filtering equipment could replace this
equipment with the cleaning equipment and still compete with new plants that
install the cleaning equipment.
doesnot say abt saving money

Plant scientists have been able to genetically engineer vegetable seeds to


produce crops that are highly resistant to insect damage. Although these
seeds currently cost more than conventional seeds, their cost is likely to
decline. Moreover, farmers planting them can use far less pesticide, and
most consumers prefer vegetables grown with less pesticide. Therefore, for
crops for which these seeds can be developed, their use is likely to become
to norm.
Which of the following would be most useful to know in evaluating the
argument above?
A. Whether plant scientists have developed insect-resistant seeds for every
crop that is currently grown commercially
B. Whether farmers typically use agricultural pesticides in larger amounts
than is necessary to prevent crop damage
C. whether plants grown from the new genetically engineered seeds can be
kept completely free of insect damage.
D. Whether seeds genetically engineered to produce insect-resistant crops
generate significantly lower per acre crop yields than do currently used
seeds.
E. Whether most varieties of crops currently grown commercially have
greater natural resistance to insect damage than did similar varieties in the
past
OA: D
IMO: B
For relevant information questions, you should figure out the assumption.
Then, you should look for a choice where if it goes one way it will strengthen
the argument, and if it goes the other way it it will weaken the argument. So,
one way to approach this type of question is to treat it as a hybrid
strengthen/weaken. Another way is to look for the choice that falls flatly
within the scope of the argument.
The author argues that genetically engineered seeds will replace
conventional seeds because they are highly resistant to insect damage,
because their cost is likely to decline, and because they require less
pesticides.
When figuring out the assumption, you should ask "what else must be true
for the argument to work?"
What else must be true in order for the genetically engineered seeds to
replace conventional ones?...well, they would have to deliver at least almost

as much yield as the conventional ones. (If, they didn't, then it is unlikely
that farmers will turn to them.)
Choice D matches this prediction. Let's apply the hybrid strengthen/weaken
test to choice D:
What if the genetically engineered seeds generated significantly lower yield?
Then, farmers are unlikely to turn to them, and the argument is weakened.
And, what if genetically engineered seeds generated just as much yield as
the conventional seeds? Then, farmers are more likely to turn to them (for all
the reasons the author cites), and the argument is strengthened.
Let's consider choices B and E.
Choice B is irrelevant. If farmers right now are using more pesticides than are
necessary, then they can simply use less pesticide; they don't have to turn to
genetically engineered seeds.
Choice E offers an irrelevant comparison. Even if current crops are more
insect-resistant than those used in the past, we still know from the passage
that the genetically engineered seeds are far more insect-resistant than the
ones being currently used. (Choice E would be wrong even if it said upcoming
conventional seeds are more insect-resistant than current ones, because we
still wouldn't know whether the upcoming ones are as insect-resistant as the
genetically engineered ones).
As an aside, it shouldn't necessarily be surprising that the correct answer
brings up an idea (yield) that did not make an appearance in the stimulus.
The right answer to a relevant information question is closely \related to the
assumption, and assumptions are always unstated.

When airlines were deregulated in the 1970s, the average price of a ticket
was $135. Three decades later, the average price is $275, there are twice as
many in air collisions, fliers in small markets are at the mercy of predatory
carriers, and air rage is at an all-time high. It is time to re-regulate the air
travel industry.
All of the following statements, if true, weaken the above argument except
for which statement?
A. Because of inflation, a $275 ticket today is actually 10 to 20% less
expensive than a $135 ticket was in 1975.
B. Deregulation has increased the choices available to fliers in terms of both
time of flight and carrier.
C. Airlines are currently flying more than five times as many passenger miles
per year as they did in 1975.
D. Compared with passengers in large urban aread, passengers in small
markets pay, on average, twice as much per mile flowin on domestic flights
because their airports are generally served by fewer airlines.
E. Psychologists have been able to determine no connection between the
deregulation of the airline industry and the onset of increased levels of air
rage.
Tutor
Conclusion: The airlines should be regulated.
Premises: The increase in price, air collisions, and air rage. Fliers in small markets
are at the mercy of predatory carriers.
Assumptions: That deregulation is causing the problems listed above. That there
are no benefits that justify the deregulation. That there are no negative effects that
would be caused by re-regulation.
We need to eliminate the 4 answer choices that weaken the conclusion that the
airlines should be regulated.

A. Because of inflation, a $275 ticket today is actually 10 to 20% less expensive


than a $135 ticket was in 1975. Shows that inflation, not deregulation, is causing
the increase in price. Weakens the causal assumption made in the argument.
Eliminate A.
B. Deregulation has increased the choices available to fliers in terms of both time of
flight and carrier. Shows that deregulation has a benefit. Eliminate B.
C. Airlines are currently flying more than five times as many passenger miles per
year as they did in 1975. Shows that an increase in the number of passengers might
be causing the problems listed in the argument. Weakens the assumption that
deregulation is causing these problems. Eliminate C.
D. Compared with passengers in large urban areas, passengers in small markets
pay, on average, twice as much per mile flying on domestic flights because their
airports are generally served by fewer airlines. Correct. Strengthens the conclusion
that the airlines should be regulated.
E. Psychologists have been able to determine no connection between the
deregulation of the airline industry and the onset of increased levels of air rage.
Weakens the assumption that deregulation is causing the air rage. Eliminate E.
The correct answer is D.

Psychotherapy has been described as a form of moral coercion. However, when


people are coerced, their ability to make choices is restricted, and the goal of
psychotherapy is to enhance peoples ability to make choices. Hence,
psychotherapy cannot possibly be a form of coercion.
Which one of the following describes a flaw in the argument?
(A) The position being argued against is redefined unfairly in order to make it an
easier target.
(B) Psychotherapy is unfairly criticized for having a single goal, rather than having
many complex goals.
(C) No allowance is made for the fact that the practice or results of psychotherapy
might run counter to its goals.
(D) The goals of psychotherapy are taken to justify any means that are used to
achieve those goals.
(E) It offers no argument to show that moral coercion is always undesirable.
OA: C
Conclusion: psychotherapy cannot possibly be a form of coercion.
Evidence:
1. when people are coerced, their ability to make choices is restricted
2. the goal of psychotherapy is to enhance peoples ability to make choices
Assumption: psychotherapy meets it goals.
Flaw: The argument fails to consider that psychotherapy may NOT reach its goals
and that therefore patients' ability to make choices may NOT be enhanced. Answer
C.

Lydia: Each year, thousands of seabirds are injured when they become entangled in
equipment owned by fishing companies. Therefore, the fishing companies should
assume responsibility for funding veterinary treatment for the injured birds.
Jonathan: Your feelings for the birds are admirable. Your proposal, however, should
not be adopted because treatment of the most seriously injured birds would
inhumanely prolong the lives of animals no longer able to live in the wild, as all
wildlife should.
Jonathan uses which one of the following techniques in his response to Lydia?
(A) He directs a personal attack against her rather than addressing the argument
she advances.
(B) He suggests that her proposal is based on self-interest rather than on real
sympathy for the injured birds.
(C) He questions the appropriateness of interfering with wildlife in any way, even if
the goal of the interference is to help.
(D) He attempts to discredit her proposal by discussing its implications for only
those birds that it serves least well.
(E) He evades discussion of her proposal by raising the issue of whether her feelings
about the birds are justified.
OA:D

The key here, as with all CR questions, is to come up with a prediction of what the right answer
should do before going to the answer choices, and only then go to the answer choices to find one
that says approximately what you had in mind.
Here, sum up what Jonathan is doing - he is giving an extreme example (probably not a prevalent
one - only the most injured birds) where Lydia's proposal can lead to more bad than good. D is
the only answer choice that says that in different words. C, the only (slightly) dangerous answer
choice, is eliminated by the extreme "in any way", which is not what Jonathan is saying at all he's presenting a specific case where interference with an injured wildlife is inadvisable. For D
to be true, Jonathan should be saying "we should not help wildlife at all, since that is interfering"
- but Jonathan limits his case only to extremely injured birds.

Das könnte Ihnen auch gefallen